Diskusní fórum teorie superstrun



Na této stránce vidíte aktuální průběh diskuse. Svůj příspěvek napište sem. Staré příspěvky a úvod do problematiky najdete tady. Vyhrazuji si právo mazat diskusní příspěvky, pokud se nebudou vztahovat k tématu nebo budou jinak narušovat průběh debaty. Rovněž nepoužívejte HTML tagy, stejně nebudou fungovat. Ještě poznámka pro ty, kteří jsou v oboru stejnými laiky jako : Nebojte se ptát. Tady funguje systém "kdo to ví, odpoví".
Předchozí stránka


Příspěvek od: mino
Čas: 20:07 10.01.2012
E-mail: mino033@centrum.sk
Text
Sú vlny pravdepodobnosti v teorii superstrún?Ako sa prejavujú?
Súvisí to s tým,či sa zvinuté dimenzie pohybujú?


Příspěvek od: Jiří Johan
Čas: 11:26 17.11.2011
E-mail: johan.jiri@centrum.cz
TextDobrý den. Rád bych si vyměnil pár názorů s p. Lubošem Motlem o jeho překladu Elegantního vesmíru. Zdvořile se ptám zda toto diskusní forum ještě funguje. Začalo 2000 a poslední příspěvek je asi z února 2011. Nemusím nutně vystupovat pro celou diskusní komunitu, ale nevyhýbam se tomu.Předmětem mého zájmu jsou všem naprosto jasné pojmy, (jak jsem si četl některé příspěvky a ostatně jak to vyplývá i z Elegantního vesmíru), jako je čas, prostor, prostoročas, atd. Prosím pana L.Motla (nebo pana správce tohoto diskusního fora) aby byl tak laskav a odpvěděl mi zde,nebo na mojí E-mail adresu.


Příspěvek od: Daniel zo Slovenska
Čas: 20:02 28.02.2011
E-mail: juwa1@azet.sk
Text
Dobrý deň,pán Motl.Ja by som Vám chcel položiť jednu otázku a dúfam,že mi odpustíte,že píšem po slovensky.
Zaujímalo by ma,či ste si už niekedy položili otázku,že možno teória Strún doteraz nepodala žiadny extra pádny argument nie kvôli tomu,že je priliš zložitá,ale jednoducho kvôli tomu,že je nesprávna.Mna strašne zaráža,čo núti mladych teoretických fyzikov,aby sa venovali práve strunám.Čo je na nich tak strašne priťažlivého,keď doteraz nepodali nijaku ani len teoretickú predpoveď? 40 rokov driny,tisíce vedeckých prác....a stále nič.....nikdy ste nemali pochybnosti? Prípadne či vôbec niekto zo strunárov má nejaké pochybnosti.To ma zaujíma.
Prajem príjemný deň.Dano


Příspěvek od: Aleš Vaníček
Čas: 19:02 20.08.2010
E-mail: hexo@seznam.cz
Dimenze,jak vy říkáte,jsou jakoby částice s miniaturním poločasem rozpadu(pro vysvětlení,či představu),ale v prostoru působení emoční enrgie,kterou superstruny zachytávají,na hmotu...


Příspěvek od: Aleš Vaníček
Čas: 18:58 20.08.2010
E-mail: hexo@seznam.cz
Teorii superstrun lze dobře aplikovat i na funkci podvědomí v lidské psychyce.Podvědomí reaguje a je živeno emocemi.Superstruny reagují na vibrace-emoce.
Dá se říci,že podvědomí spolupracuje,či využívá přímo superstrun k úpravě tvaru hmoty či sousledu.
Vesmír je stavebnice a emoce jsou pro nás klíčem ke stavění.
Je to všechno založené na spolupráci.Vědomí a podvědomí.
Co si o tom myslíte?


Příspěvek od: Shakti
Čas: 21:37 27.01.2009
E-mail: monucerna@hotmail.com
Ráda bych věděla, jak se struny, které jsou "namotané" na svinutou /kruhohovou/ prostorovou dimenzi na tuto dimenzi vlastně namotají.
V teorii superstrun se automaticky předpokládá, že to jde a mě to do hlavy nejde.
Dík za odpověď.
Shakti


Příspěvek od: Ladislav
Čas: 17:53 29.08.2008
E-mail: lada.vondrasek@email.cz
Jak může být teorie superstrun správná,když ve vesmíru evidetně funguje princip dekoherence a nevypadá to,že vesmír je interakčně tak extrémně propojený,jak o to dává výklad Teorie superstrun.To by byl celývesmír propleten změtí superstrun a veškerá hmota by tu byla ovlivněná i třeba minimální interakcí nějaké hodně vzdáleného tělesa.Prostě je ta teorie nedokončená a tejí výklad je zavádějící.


Příspěvek od: Standa Hampl
Čas: 16:42 02.03.2008
E-mail: plikous@email.cz
Zdravim, v roce 2000 jsem sem ještě jako student střední školy psal pár příspěvků... Dneska mám už po vejšce a musím přiznat, že teorii superstrun se již nevěnuji a vůbec mě to nemrzí:-) Nicméně fandím všem, kteří u toho vydrželi.


Příspěvek od: Petr Nový
Čas: 19:07 26.12.2007
E-mail: novy@pentre.cz
Prolistoval jsem tuto diskusi od samého začátku a musím konstatovat, že ty tam jsou odborné a pro laiky zcela nesrozumitelné argumentace pánů Motla a Brože. Diskuse běžela v té době na plné obrátky. Zato nyní zkomírá a je to velká škoda. Dovolil bych si proto dotaz přímo na pana Motla: Dokážu připustit, nikoli však představit, že jakýkoliv objekt může kmitat v deseti rozměrech v čase, který je reprezentován rozměrem jedenáctým. Nedávno zde padlo vysvětlení, že struna je jen jedna a to, co jsme dosud považovali za různé elementární částice, je pouze projevem různým pohybových stavů této jedné struny. Je to tedy jako s teplotou, kterou někdy vnímáme jako příjemné teplo, jindy jako horko nebo zase jako chlad, ale přitom se jedná vlastně o kmitání molekul s různou frekvencí a energií. A teď tedy k totazu: Pochopil jsem to správně, že každá struna je desetirozměrný objekt, který ale nemusí kmitat vždy ve všech deseti rozměrech?
Petr


Příspěvek od: Martin Pacák
Čas: 01:10 02.12.2007
E-mail: s@anasat.cz
Rád jsem tady před pár lety listoval. Děkuji Vojtovi


Příspěvek od: egg
Čas: 18:00 03.12.2006
E-mail: egg@matfyz.cz
Změna! Ponechal jsem možnost psát příspěvky, ale vybavil jsem to malým Turingovým testem. Člověk dokáže vyplnit obsah políčka heslo, protože si ho hned vedle přečte. Spamerský program bude mít problém.


Příspěvek od: egg
Čas: 16:17 03.12.2006
E-mail: egg@matfyz.cz
Diskusní fórum jsem dnes uzavřel. Důvody jsou dva: množství spamu, které nestíhám odstraňovat, a také fakt, že diskuse je prakticky mrtvá. Nikdo kvalifikovaný tu posledních pár měsíců neodpovídal. Zájemce odkazuji na moderované diskusní fórum serveru Aldebaran, kterého se účastním i já.
http://aldebaran.cz/forum/
Není úzce zaměřené na superstruny ale obecně na fyziku.

Využívám tohoto posledního příspěvku, abych aspoň podle svých současných vědomostí odpověděl na poslední položené otázky.

milan 12:30 19.09.2006: Jak by se projevovala, normálně. :-) Je zprostředkována strunami, které odpovídají částicím interakce. EM síla fotony, gravitační síla gravitony.

vivo 14:33 10.09.2006: Není. Brian Greene v poslední knize Struktura vesmíru zmínil, že byly naměřeny jisté odchylky od Newtonova gravitačního zákona na vzdálenostech v řádu desetin milimetru. To by s největší pravděpodobností znamenalo existenci dodatečných dimenzí této velikosti a tedy potvrzení superstrunové teorie. Nicméně, jak poznamenává i Greene, takové mimořádné tvrzení je třeba dokázat mimořádně spolehlivě, takže pokusy budou opakovány za důsledného vyloučení vnějších vlivů apod. Teprve tím se objev potvrdí či vyvrátí. Informace v knize může být zastaralá, já novější nemám, ale řekl bych, že v případě potvrzení bychom se o tom všichni dočetli, byla by to bombastická zpráva.

Karel Sedláček 13:38 01.01.2006:
1) V podstatě ano, ale je to labilní rovnováha, takže je IMHO prakticky nemožné nastavit to tak trvale. Může obíhat černou díru jako družice Zemi, ale kvůli extra krátké době oběhu se sebemenší odchylka od ideální dráhy rychle projeví. To máte jako na skleněné desce stavět jehlu na špičku.
2) Moc tomu nápadu nerozumím, zkuste to nadhodit na Aldebaranu. Jinak ale na gravitační pole nejsou potřeba gravitony, statické pole je tvořeno jen virtuálními gravitony. Skutečný graviton představuje *změnu* gravitačního pole, takže jimi jistě neuděláte pole homogennía časově stabilní.
3) Pokud vím, tohle vyplývá z matematiky M-teorie. Ale neznám ji. Něco o těchto výsledcích je také ve Struktuře vesmíru.

Wladimiro 17:10 16.09.2005: Tak docela správné to není, struny jsou pořád něco, co vibruje *v prostoru*. Pokud vím, není to vibrující *sám prostor*.


Příspěvek od: milan
Čas: 12:30 19.09.2006
E-mail: motl@feynman.harvard.edu
Ako sa v teorii strun prejavuje prirodna sila,napr.gravitacna,alebo elektromagneticka?


Příspěvek od: vivo
Čas: 14:33 10.09.2006
E-mail: motl@feynman.harvard.edu
Je uz experimentalne potvrdena teoria superstrun.


Příspěvek od: milan
Čas: 13:56 10.09.2006
E-mail: fabinger@stanford.edu
Ake su pokroky v teorii superstrun a v M-teorii.


Příspěvek od: Karel Sedláček
Čas: 13:38 01.01.2006
E-mail: karel.post@post.cz
O vánocích jsem si po přečtení diskuse na tomto foru koupil a přečetl Elegantní vesmír a byl jsem překvapen jak lehce k pochopení jsou zde jednotlivé věci presentovány. Po přečtení knihy bych měl ale několik otázek na které mě napadli :

1. Za předpokladu že se foton dostane na horizont událostí, může to znamenat že bude de facto obíhat kolem černé díry?

2. V případě, že bude někdy v budoucnu možné sestrojit, je to spíše otázka ze žándru sci-fi, nějaký ,,emitor gravitonů,, a samozřejmě bude existovat něco čímž neprojdou tak, aby mohl tento emitor vytvářet extrémně silné a zároveň i homogenní gravitační pole, tak díky extrémní hustotě gravitonů zde může být vytvořena dokonalá stáze pro jakýkoliv předmět uvnitř oproti plynutí času okolí?

3. Když jsem se v Elegantním vesmíru od B. Greena dočetl, že dle M-teorie má náš vesmír patrně 11 rozměrů, 1 časový a 10 prostorových, tak mě napadá, jak se přišlo na to že nutně musí být jen jeden časový a zbylých 10 prostorových?

Chápu že když pozorujeme 1 časový a 3 prostorové, že ostatní rozměry jsou patrně svinuté v Calabiho-Yauovy variety, ale kde je dáno že to musí být zrovna prostorové a navíc co kdyby tomu bylo i jinak. Na str. 52 v El. Vesmíru jsem se dočetl, že dle Einsteina se všichni pohybujeme rychlostí c v čase, díky tomu všichni vidíme vesmír z našeho pohledu a zdá se nám to logické. Jak by ale vesmír vypadal z jiného pohledu? Třeba takového fotonu?

Foton o nás nic neví. Z jeho pohledu má vesmír jen jediný prostorový rozměr a to vpřed nebo vzad, a n rozměrů časových. Když uběhne dle vlastního času fotonu např. sekunda (připusťme to), tak foton urazil de facto nekonečnou vzdálenost v jeho jediném prostorovém rozměru. Z našeho pohledu ne, ale z jeho pohledu to vypadá jinak. Foton nikdy nebude vidět více prostorových rozměrů, jelikož z jeho pohledu mu budou skryty, ale např. může vidět více časových, asi pro něj nebude problém jít zpět v čase či dopředu či dokonce v čase vlevo, vpravo, nahoru či dolů, z jeho pohledu je to normální věc, ale bude problém jít zpátky v tom jeho jednom prostorovém rozměru. Kdyby např. měl kamaráda fotona co najednou umřel a chtěl by ho zachránit, tak pro foton by nerozhodovalo kdy kamarád zemřel, ale kde. Fotonu by pak stačilo jen zastavil a letět na opačnou stranu, ale kdyby to udělal, tak jak by začal zpomalovat najednou by zjišťoval že prostorových rozměrů je více a že už nelze jít libovolně sem a tam v čase. Díky tomu by jeho vlastní svět jak ho znal jakoby utekl rychlostí c v prostoru a foton by ho už nikdy nemohl dohonit a kdyby se dostal do prostoru kde jeho kamarád foton zemřel, tak by to byl již úplně jiný prostor.

Z pohledu našeho by pak mohla být analogie v cestování v čase, jakmile bychom chtěli zpátky v čase, tak bychom museli čas zpomalit a zastavit a pak letět zpátky, ale jakmile bychom to udělali a úplně zastavili, tak by naše okolí jakoby v čase uletělo rychlostí c a za náš vlastní čas jedné sekundy by vesmír možná ani neexistoval a až v tuhle chvíli bychom se mohli otočit zpět a vrátit v čase tam kde by už bylo něco úplně jiného.

A rozdíl je jen v tom že my máme rychlost c v čase a foton v prostoru, pro nás není problém jít vpřed či vzad, ale v čase letíme jen vpřed, foton to ale může vidět naopak, není problém pohybu v čase, ale v prostoru. Tohle je jen můj vlastní názor a vzhledem k tomu že se fyziku zajímám jen jako o koníčka nevím zda to není třeba jen úplný nesmysl.

Díky
Karel


Příspěvek od: Wladimiro
Čas: 17:10 16.09.2005
E-mail: Dobrev@seznam.cz
Ahoj není to tak dlouho co jsem se trochu seznámil s teorií superstun a tak si chci ověrit pravdivost svých domněnek.
Podle teorie superstrun je tedy všechna hmota vesmíru tvořena zakřivením časoprostoru - když si tedy představíme prostor
jako tkaninu (třeba tričko nebo mikinu) pak veškerá hmota jsou přehyby na této tkanině(zmuchlaní)? To by potom znamenalo že není ani čistě hypoteticky možné cestovat do jiných vesmírů(nejde přece přenést zmuchlání z jednoho kusu hadru na druhý)?

P.S.Jsem jenom laik a takto si tuto teorii představuji a byl bych rád kdyby někdo mé domněnky uvedl na pravou míru.


Příspěvek od: Eva
Čas: 18:07 05.05.2005
E-mail: eva.hasova@seznam.cz
Něvěděl byste prosím Vás někdo, jak se odvodí Planckova konstanta? Předem díky.


Příspěvek od: rasto
Čas: 13:47 29.04.2005
E-mail: rasto.danko@azet.sk
Text
Ok vibracie omega x strun, ktore su v lamda chi priestoroch homeomorfne s n(i) rozmernymi M branami sa daju matematicky lahko popisat hypergeometrickymi funkcami, ktore vytvaraju na Kummerovej imaginarnej ploche nekomutativnu Poincareho grupu 11. radu.
Relacia ekvivalencie definovana na specialnej podvariete Kummerovej plochy rozlozi k-rozmerne kontinuum na subsystem singularit t.j. izolovanych bodov, ktorych mohutnost je zhodna mohutnosti Cantorovho diskontinua a ktoreho fraktalna dimenzia lezi v intervale 1.45, 1.8097.
Vysledne objekty maju Lebesguovu mieru rovnu nule, avsak delta transformaciou je mozne uskutocnit ich kompaktifikaciu na Hilbertov priestor nenulovej miery, co umoznuje vysvetlit dimenzionalnu koordinaciu a tym vysvetlit preco vsetky oblasti priestoru a teda aj vsetky M brany a superstruny pri paralelnom prenose maju vsetky resp. zakladne charakteristiky invariantne.


Příspěvek od: Láďa
Čas: 11:04 15.12.2004
E-mail: tuna@muench.cz
I lidový myslitel může přinést podnět.
Ano superstruny jsou stejné ale dle mého názoru "kmitají" 24 způsoby (12+12 symetrických). K tomu, že jsou časového původu bych chtěl přistoupit pomaleji abych nebyl okamžitě za blázna.
K mému dotazu o supersymetrii času není názor? Přez toto bych svou myšlenku mohl vysvětlit blíže.





Příspěvek od: Vojta Hála
Čas: 10:29 15.12.2004
E-mail: egg@matfyz.cz
Láďo, superstruny jsou dle teorie všechny stejné, jedna jak druhá, jen různě kmitají.

Co znamená, "jsou časového původu"?? Ale upozorňuju, že pro teorie lidových myslitelů tu není místo.


Příspěvek od: Láďa
Čas: 10:02 15.12.2004
E-mail: tuna@muench.cz
Text
Kolik je tedy vlastně základních částic (superstrun). Domnívám se, že jich je 24 a jsou časového původu.Zabývá se někdo supersymetrii času?
Děkuji za odpověď.
Láďa


Příspěvek od: Jan Werner
Čas: 13:59 15.10.2004
E-mail: darkpilgrim@seznam.cz
Diky za rychlou a kompletni odpoved. Byl jsem si vedom, ze to asi nebude mit primy vztah k tematu fora, ale riskl jsem to, protoze mi diskuze pripadala erudovana a tak elementarni otazka by nezatezovala nebo nesvadela diskuzi od tematu.

Jeste jednou diky;-)...


Příspěvek od: Vojta Hála
Čas: 17:48 14.10.2004
E-mail: egg@matfyz.cz
Web: http://hyperkrychle.cz
1. Klidová hmotnost je hmotnost objektu v klidu. Foton nikdy v klidu není, pohybuje se vždy konstantní nenulovou rychlostí c. Takže pojem klidová hmotnost pro něj nedává dobrý smysl. Nicméně pokud trváme na tom, že ji chceme i pro foton definovat, musí to být jedině nula. Proč? Relativistická hmotnost hmotných těles roste s rychlostí podle vztahu

m = m_0 / sqrt(1-(v/c)^2),

kde m_0 je klidová hmotnost, v je rychlost. Pokud m_0 je nenulové číslo, vidíme, že když se v přibližuje k c, tak m roste nade všechny meze. Pro v=c vychází nekonečná relativistická hmotnost, což zjevně není pravda. Takže pro foton pohybující se rychlostí c nemůže mít nenulovou klidovou hmotnost. A stejná argumentace platí i pro jakékoliv jiné částice pohybující se rychlostí c.

2. Ano, se dvěma poznámkami. Jednak rychlost fotonu ve vakuu je konstanta. (O hmotném prostředí se teď doufám nebavíme.) A jednak klidně můžete mluvit o úměře. Energie fotonu je
E = hf,
kde h je konstanta a f je frekvence. A zároveň je
E = m . c^2,
kde c je konstanta a m hmotnost. Takže
m = hf/c^2,
přičemž h/c^2 je konstanta úměrnosti.

3. Ne, foton v klidu neexistuje. Foton s nulovou frekvencí taky ne, protože by se nijak neprojevoval - měl by nulovou hmotnost, energii i hybnost.

Opět ale musím poznamenat, že tohle jsou elementární otázky z kvantové mechaniky a speciální relativity. Takže na jejich zodpovězení nijak nepotřebujeme superstruny a tudíž se o nich nemusíme rozepisovat tady. Jako v předchozím příspěvku Vás odkazuji na fórum Aldebaran, kam by takové otázky naopak patřily.

Mějte se fajn!


Příspěvek od: Jan Werner
Čas: 15:21 14.10.2004
E-mail: darkpilgrim@seznam.cz
Dobry den,

jsem ve fyzice a matematice pouze laik, nicmene po zbeznem prolistovani tohoto zajimaveho fora dospel k nekolika otazkam:

1. Je klidova hmotnost fotonu opravdu nulova nebo nenulova?
2. Vztah mezi hmotnosti, rychlosti a frekvenci fotonu zapricinuje promenlivou hmotnost fotonu, zjednodusene receno se vzrustajici frekvenci lze povazovat hmotnost fotonu za zvysujici se (umyslne nevkladam umeru)?
3. Existuje vubec foton v klidovem stavu s nulovou frekvenci, nebo jde pouze o imaginarni stav?

Doufam, ze nebudu odsuzovan pro formulacni i obsahovou jednoduchost svych otazek:-) a snad mi je nekdo v tomto smeru erudovany pomuze objasnit, mam jistou komplexni predstavu, nicmene jako opravdovy laik nejsem presvedcen o jeji spravnosti. Predem diky.


Příspěvek od: Vojta Hála
Čas: 01:08 03.10.2004
E-mail: egg@matfyz.cz
Web: http://egg.matfyz.cz
Měl jsem zato, že diskuze tu definitivně usnula, ale zjevně ji lidi čtou, to mám radost! :-) Pro nestrunové otázky můžu doporučit fórum na serveru Aldebaran.
http://aldebaran.cz/forum/


Příspěvek od: Jan Olšina
Čas: 00:56 03.10.2004
E-mail: irigi@matfyz.cz
Web: irigi.matfyz.cz
Vážení přátelé,
mám fyzikální dotaz, který přímo s teorií strun nesouvisí, spíše s obecnou relativitou a kvantovou mechanikou, ale jelikož jsem nenašel žádné jiné vhodné diskusní fórum (prosím doporučte mi nějaké, děkuji!), ptám se zde:

Představme si černou mikrodíru vhozenou do kapaliny (dejme tomu vody) - v její bezprostřední blízkosti na kapalinu působí její gravitační síla a tedy je zde voda pod velmi vysokým tlakem, až v určité vzdálenosti kapalina nějakým způsobem ztratí svou strukturu a od určitého poloměru dále padá volným pádem do středu. K čemu vlastně na tomto poloměru dochází? Dojde napřed k rozpadu molekul vody až postupně k neutronové degeneraci? (jako např. u neutronových hvězd?) Jaká bude přibližně velikost tlakové síly, pro kterou k tomuto jevu u běžných kapalin dojde?

Předem děkuji,
přeji hezký den!


Příspěvek od: Pavel Bednář
Čas: 23:28 01.06.2004
E-mail: zorl@seznam.cz
Když jsem četl o teorii superstrun (mluvím i o M-teorii), vždy jsem dozvěděl, že počet rozměrů vesmíru je nějaké číslo. Dříve to bylo 10, teď díky známým dualitám je to 11 rozměrů.
Není možná existence ještě dalších rozměrů, které třeba nemají na náš vesmír v rámci teorie vliv a tím jsou jaksi nezjistitelné?


Příspěvek od: Luboš Motl
Čas: 15:58 19.06.2003
E-mail: motl@feynman.harvard.edu
Dobrý den, nevím, jestli je to otázka na mě, ale já bych s Vámi souhlasil. Energie je jedno číslo, které se zachovává jako důsledek symetrie fyzikálních zákonů vůči posunu v čase. Má různé příspěvky (formy), kinetickou, potenciální, chemickou apod., v závislosti na aproximaci, se kterou popisujeme ten či onen systém. Různé formy energie lze přeměňovat na sebe navzájem, byť v určitém směru to může být snazší než ve směru opačném. Proto říkáme, že jistá energie je lépe využitelná - jako například chemická energie v benzínu - zatímco jiná energie (zvláště tepelná) je prakticky "ztracená". V teorii strun je veškerá energie a hmota složena ze strun, jejich množství a hlavně pohybového stavu. Také nemám rád, pokud se slovo "energie" používá pro všemožné abstraktní věci.


Příspěvek od: laik
Čas: 14:55 19.06.2003
E-mail: laik@laik.cz
Ahoj,
v úvodu do úvodu zmiňujete,že jednorozměrné vlákno energie strunou sluje.
Vždy jsem na energii nahlížel jako na kouzelné číslo,které ve své podstatě vypovídá o stavu sytému a schopnosti systému měnit svůj stav(pohybový,tepelný..),z vámi uváděného však plyne možnost,že by energie mohla mít i tvar,v případě struny měřitelný jen jedním způsobem(jednorozměrný),což mi jaksi nejde pod fousy a tak budu potěšen,vysvětlíte-li mi jak to bylo vlastně míněno,či v čem si to špatně vykládám.

Děkuji za odpovědi
Laik


Příspěvek od: Luboš Motl
Čas: 13:22 05.05.2003
E-mail: motl@feynman.harvard.edu
Milý pane Bednáři, vazebná konstanta je spojitá veličina, a tak je nekonečně málo pravděpodobné, že se člověk trefí přesně do bodu jedna. Spíše ho o kouseček mine, tam či onam.

Pokud se přesto trefí, tak poruchové součty buď divergují, nebo konvergují mnohem pomaleji než pro jakoukoliv menší hodnotu vazebné konstanty. Vezměte si třeba řadu 1+q+q^2+q^3... Ta konverguje jen v případě, že je q menší než jedna.

Na druhé straně řada 1-q+q^2/2-q^3/3+q^4/4-q^5/5... konverguje v případě, že je q menší nebo rovno jedné.

Spíše bych v tomto kontextu chtěl upozornit na to, že Brian položil hranici do bodu jedna v zájmu jednoduchosti, ale ne vždycky tomu tak je. Často se vazebná konstanta g volí tak, že poruchové součty konvergují, je-li g menší než nikoliv jedna, ale odmocnina ze čtyř pí a podobně.

Takové věci člověk musí pochopit, pokud počítá konkrétní číselné výsledky, ale podstatu věci asi tyto detaily neovlivňují.

Hezký den přeje
Luboš


Příspěvek od: Pavel Bednář
Čas: 10:24 05.05.2003
E-mail: zorl@seznam.cz
Nedávno jsem četl knížku od Briana Greena Elegantní vesmír. Byla to výborná kniha a skvělý překlad.
Mám ale jednu otázku:
V knize se řešily případy, kdy vazebná kostanta je větší, nebo menší než jedna. Co se ale stane , když je vazebná konstanta rovna jedné? Jak lze v tomto případě použít poruchovou teorii?

Předem děkuji
Pavel Bednář


Příspěvek od: Lubos Motl
Čas: 17:49 21.04.2003
E-mail: motl@feynman.harvard.edu
Web: http://lumo.matfyz.cz/struny/
Melissa Chiuova, spoluFellowkyne ze Society a matematicka, o tomto vysledku hodne mluvi - je to skutecne velka udalost na poli dnesni matematiky.

Jan Kapoun si v tom clanku asi popletl homotopii s holotropnim dychanim - jinak asi nejde vysvetlit, ze do sluvka "homotopie" systematicky pridava pismenko "r".

Kazdy matematicky vysledek podobne dulezitosti teoretika musi potesit, ackoliv o primych aplikacich nevim. Fyzici vetsinou hovori o trochu konkretnejsich varietach, jejichz topologie vcetne homotopie je znama. V zadnem tomto pripade samozrejme nebyla Poincareho domnenka narusena. Domnenka je o tom, ze neexistuji ani jine, hypoteticke pripady, kde by homotopie nestacila. Fyzici se ale malokdy zajimaji o hypoteticke variety, ktere nedokazou ani definovat, a "schopnost popsat veskerou topologii pomoci homotopie" neni prave typickym prikladem fyzikalni otazky.

Zjednodusene o domnence: homotopie udava informaci o tom, kolik druhu kulovych ploch ruzne dimenze (a kruznic), ktere neni mozne stahnout do bodu, lze vnorit do daneho tvaru. Kuprikladu na d-rozmerne sfere lze kazdou menerozmernou sferu (krome 0-rozmerne a d-rozmerne) smrsknout do bodu. Otazkou je, zda kazde d-rozmerne teleso (varietu), na nemz (na niz) lze kazdou sferu o dimenzi 1 az d-1 smrsknout do bodu, je topologicky ekvivalentni sfere, tedy zda ho lze plynule do sfery pretransformovat. Poincareho domnenka tvrdi, ze ekvivalentni je a ze zadna dalsi telesa tohoto typu nejsou.

Zdravi
Lubos


Příspěvek od: Vojta Hala
Čas: 16:58 21.04.2003
E-mail: egg@matfyz.cz
Web: http://egg.matfyz.cz
Vsem vesele Velikonoce! :-)

http://www.scienceworld.cz/sw.nsf/ID/3533DB3808C70536C1256E970048FB1D

Co znamena Poincareho hypoteza pro teorii superstrun? Jsou nejake nove informace od nedavne doby, kdy o ni psal Keith Devlin ve sve (mimochodem velmi zdarile) knizce? Byl Perelmanuv dukaz spravny nebo ne? Dostal ten milion dolaru? A co na to Jan Tleskac? ;-))


Příspěvek od: Luboš Motl
Čas: 13:08 11.04.2003
E-mail: motl@feynman.harvard.edu
Pane Bednáři, teorii z roku 1999, postavenou na "proměnnosti rychlosti světla c", asi nechám někomu jinému, protože bych pro ni nejspíše dobrého slova nenašel. Rychlost světla ve vakuu je už podle definice jednoho metru konstantní. Ale jinak plochost vesmíru - tedy velmi malé zakřivení prostorového průřezu vesmíru - vysvětluje inflační teorie, kterou Alan Guth poprvé představil v roce 1979.


Příspěvek od: Pavel Bednář
Čas: 09:34 11.04.2003
E-mail: zorl@seznam.cz
Děkuji ( i když opožděně ) za objasnění mých otázek.
Mám ale další otázku:
V rámci technických možností bylo zjištěno, že náš vesmír se vyskytuje někde na hranici mezi otevřeným a uzavřeným vesmírem, čemuž se vědci diví, protože tato situace by měla být nestabilní. A teď k věci:
Slyšel jsem, že existuje teorie (měla vzniknou někdy na přelomu 1999/2000?), podle které se vesmír vyvíjí právě tak, aby byl na rozhraní mezi otevřeným a uzavřeným vesmírem. Mělo by to nějak souviset s energií vakua. Jedním z důsledků by pak mělo být i to, že rychlost světla ve vakuu c by nebyla konstantní.
Co je na tom pravdy?


Příspěvek od: Ka-mila
Čas: 16:37 05.04.2003
E-mail: plevanka@seznam.cz
Text
Děkuji moc za objasnění:-))).Četla jsem knížku od Briana Greena, skvělý překlad... Akorát trošku záhul na mojí mozkovnu! Moc vás zdravím. Kamila


Příspěvek od: Luboš Motl
Čas: 14:13 03.04.2003
E-mail: motl@feynman.harvard.edu
Web: http://www.matfyz.cz/lumo
Ahoj Kamčo,

dělení dvojhvězd na tyhle tři skupiny probíhá podle toho, jak snadno můžeme dvojhvězdu identifikovat - v podstatě podle toho, jak jsme poznali, že jde o dvojhvězdu. Oběh *vizuálních* dvojhvězd lze přímo sledovat velkým dalekohledem a trvá několik let nebo i staletí. *Spektroskopické* hvězdy už přímo vizuálně nerozlišíme, ale pokud si uděláme spektrální rozklad jejich světla, tak zjistíme, že se každá spektrální čára rozdvojuje na dvě čáry, které se k sobě přibližují a zase vzdalují v rytmu obíhání dvojhvězdy. To se děje kvůli Dopplerovu efektu: od vzdalující se hvězdy přijímáme menší frekvenci než od hvězdy, která se právě přibližuje. Hvězdy nazýváme zákrytové, pokud jedna druhou alespoň částečně zakrývá v určité fázi obíhání. Na to musí být rovina oběhu dostatečně přesně rovnoběžná ke spojnici Země a dvojhvězdy. V takovém případě lze měřit úbytek celkového osvětlení v momentu, kdy se překrývají.

Zdraví
Luboš


Příspěvek od: Ka-mila
Čas: 14:00 03.04.2003
E-mail: plevanka@seznam.cz
TextDobré odpoledne, můj problém tkví v dvojhvězdách...No ani nevím zda je vhodné se ptát zde, ale pochopte mě- matfyz mě čeká(nejspíš)až příští rok. Vím že dvojhvězdy obíhají vlivem své přitažlivosti po eliptických drahách kolem společného těžiště, ale už né tak jasné je mi jejich rozlišení na vizuální, spektroskopické a zákrytové...? No snad nikoho nerozčílím tím, že to nevím:-)! Snad mi někdo odpoví...Zdraví Kamča;-)


Příspěvek od: Vojtěch Hála
Čas: 21:18 02.04.2003
E-mail: egg@matfyz.cz
Web: http://egg.matfyz.cz
Pane Michálku, nezlobte se, ale tahle diskutárna se má věnovat mainstreamu v teoretické fyzice a teorii superstrun a nikoliv teoriím jednotlivců. Proto Vás prosím, abyste si věci ohledně Vaší teorie vyřizoval s kompetentními lidmi e-mailem a nikoliv zde. O totéž prosím ty, kteří mají případně chuť na Vaši výzvu reagovat.


Příspěvek od: Ladislav Michálek
Čas: 21:00 02.04.2003
E-mail: michalek.l@worldonline.cz
Web: http://sweb.cz/mchlk/Fotony.htm
Text
Na výše uvedeném webu uvádím svou teorií o fotonech v souvislosti s možným vysvětlením konstantní rychlosti světla. Prosím o námitky a posouzení.


Příspěvek od: Luboš Motl
Čas: 15:44 01.04.2003
E-mail: motl@feynman.harvard.edu
Milý pane Bednáři,
omlouvám se za předchozí zkrácení Vašeho příjmení.

Poctivě řečeno, ono se toho v částicové fyzice zas tak příliš moc neděje. Větší objevy v experimentální částicové fyzice očekáváme až někdy v roce 2007, kdy se rozběhne urychlovač LHC. Nechci předbíhat.

V M-teorii se děje ledacos, byť jsou lidé ohledně nejnovějšího vývoje méně optimističtí, než před pár lety. Řada nových modelů naznačuje, že existuje velké množství vakuí, a proto lidé možná budou muset přemýšlet alespoň trochu v rámci antropického principu. Lidé studují časové závislá pozadí; následují Dijkgraafa a Vafu s jejich technologií výpočtu gluinového superpotenciálu pomocí maticových modelů. A tak podobně. Bylo by to asi na delší vyprávění.

Oscilace neutrin. Spin se samozřejmě při oscilacích zachovává. Velikost spinu neutrin je 1/2 a zachovává se i směr momentu hybnosti. Leptonové číslo se obecně zachovávat nemusí. Rozhodně se nezachovávají jednotlivá leptonová čísla "elektronové", "mionové" a "tauonové", ale zpracování experimentálních dat vede k výsledku, že ani celkové leptonové číslo není zachováváno při oscilacích. Oscilace neutrin pochází z "hmotnostní matice" neutrin. Mimodiagonální položky této matice odpovídají za přeměnu jednoho typu neutrina na jiný typ. Hmotnost je podle kvantové teorie pole jakýsi "kvadratický člen" ve vzorci pro celkovou energii systému. Tento kvadratický člen je schopen zničit částici a zase ji vytvořit, a takový proces je podle kvantové teorie pole zodpovědný za nenulovou klidovou hmotnost částice.

Hmotnostní matice neutrin mohou být dvou druhů: Diracova hmotnostní matice a Majoranova hmotnostní matice. Diracovu hmotnost může mít neutrino jen v případě, že existují také "pravotočivá neutrina" - tedy množství polí (druhů) neutrin je dvakrát větší, než se zdá na první pohled. V takovém případě lze rozlišit neutrino od antineutrina - zcela analogicky, jako lze rozlišit elektron a pozitron. Diracova hmotnost neutrina není v rozporu se zachováním leptonového náboje. Pokud mají neutrina Diracovy hmoty, v podstatě jsou stejným druhem částic jako elektron, jen jsou mnohem lehčí.

Druhým typem hmotnosti - a podle experimentů se zdá, že v přírodě relevantním - je hmotnost Majoranova. Tyto hmotnostní členy dávají neutrinu "váhu" tak, že ho přeměňují na jeho antineutrino a zase nazpátek. Při takovém procesu se změní leptonové číslo o 2 nebo -2, každopádně je zákon zachování hmotnosti narušen. Z druhé strany nemá v tomto případě cenu rozlišovat neutrino od antineutrina.

Gluonů je osm, nikoliv devět. Jak správně říkáte, symetrie silné interakce je SU(3) a má dimenzi osm (protože determinant musí být jedna), a nikoliv U(3), kterážto grupa má dimenzi devět. V řeči barevné-antibarevné, existuje, jak správně říkáte, devět kombinací a priori. V kvantové teorii tvoří těchto devět možností devítirozměrný lineární vektorový prostor. Ovšem existuje jeden směr v tomto prostoru - říkáme mu "diagonální" směr - a tento směr je přímo spojen s oním determinantem - který s kvarky vůbec neinteraguje.

Tímto "směrem" v devítirozměrném prostoru barev-antibarev je homogenní směs DET=(zelená/antizelená+červená/antičervená+modrá/antimodrá). Když si tuto kombinaci vyjádříte jako tabulku 3x3, přirozeně dostanete jednotkovou matici (jednotky na hlavní diagonále, jinak nuly.) Pokud si představíte gluon této smíšené "dvojbarvy", zjistíte, že je naprosto necitlivý na barvy kvarků. Pokud je kvark zelený a srazí se s gluonem DET, zůstane zelený. Pokud je červený, zůstane červený. Pokud je modrý, zůstane modrý. Důsledkem toho, tato kombinace DET vůbec s ničím neinteraguje. Ani s kvarky, ani s gluony. Toto tvrzení je přímým protějškem faktu, že grupa U(3), která má dimenzi devět, je vlastně tatáž grupa jako SU(3) x U(1), přičemž SU(3) má dimenzi osm a U(1) má dimenzi jedna, ovšem U(1) se vším "komutuje" - tedy pro libovolný prvek C z U(1) a libovolný obecný prvek D z U(3) platí CD=DC. Komutovat znamená ve fyzice kalibračních teorií "neinteragovat". Jak jistě sám vidíte, U(1) je diagonální část, která v podstatě odpovídá determinantu původní matice z U(3). Jinak řečeno, vůbec žádná částice není nabitá vůči této "diagonální" grupě U(1), a tudíž s touto barvou DET gluonů vůbec nic neinteraguje. Gluon této barvy je nefyzikální a fyzici ho proto zcela vynechávají.

S pozdravy
Luboš


Příspěvek od: Pavel Bednář
Čas: 10:47 01.04.2003
E-mail: zorl@seznam.cz
Dobrý den.
Co se týče těch nejnovějších tendencí ve fyzice, tak jsem přemýšlel spíše o částicové fyzice, a dále o m-teorii.

Když už sem ale píšu, chtěl bych se zeptat na pár věcí:

1. Relativně nedávno byla dokázána oscilace neutrin. Jak tato oscilace probíhá ( např. co se dějě se spinem, s leptonovým číslem a podobně )? Jak je to pak s tím, že neutrino je totožné se svým antineutrinem (když to ještě navíc konfrontujeme s leptonovým číslem)?

2. Mám problém pochopit gluony. Už mnohokrát jsem četl, že gluon lze chápat jako částici, která je složená z barvy a antibarvy (když se přeměňuje červený kvark na modrý, vyzáří se červená+antimodrá, což lze ztotožnit s gluonem). Když jsem ale takto kombinoval barvy, dostal jsem počet částic, jako násobek 3. Když vezmeme možnost pouze barva-anti(jiná barva)-např modrá-antizelená, dostaneme počet gluonů 6. Pokud vezmeme navíc možnost barva-anti(stená barva) např. zelená-antizelá, dostaneme počet gluonů celkem 9. Nikdy jsem ale nedostal počet gluonů 8.
Vím, že se v rámci silné interakce se počítá s SU(3) symetríí, kterou popisují komplexní unitární matice , jejichž det=1, z čehož pak plyne 8 volných parametrů a tedy i 8 gluonů. (Sám jsem si to zkusil :-).)
Problém je, že kombinatorické hledisko mi nedává stejné výsledky.
Kde je v mém uvažování chyba?
Prosím, berte ohled na to, že jsem pouhopouhý laik, který si někde někdy neco přečte, a pokud to jde, tak to vysvětlete polopatě.


Příspěvek od: Luboš Motl
Čas: 20:54 31.03.2003
E-mail: motl@feynman.harvard.edu
Web: Tendence
Omlouvám se, ale "nejnovější tendence ve fyzice" je příliš široký pojem. Nemohl byste být trochu přesnější?


Příspěvek od: Pavel Bednář
Čas: 20:48 31.03.2003
E-mail: zorl@seznam.cz
Moc děkuji za odpověď. Rád bych se ale dozvěděl o daném tématu více. Neměl by jste nějakou radu ohledně literatury (nejlépe na internetu), která by byla dostupná a vysvětlovala nejnovější tendence ve fyzice i laikům (jako například mě)?


Příspěvek od: Vojtech Hala
Čas: 16:46 31.03.2003
E-mail: egg@matfyz.cz
Web: http://egg.matfyz.cz
Lumo wrote: "Pane Bedne..."
Omlouvam se, zkomoleni jmena Bednar nezpusobil Lubos Motl nybrz technicka chyba, protoze se spatne konvertuje ceska diakritika z prispevku do odesilaneho e-mailu. Opravim to az bude cas a budu vedet jak. Zatim se tomu lze vyhnout psanim prispevku bez diakritiky a take ctenim prispevku primo z webu, kde by mela byt diakritika v poradku.


Příspěvek od: Luboš Motl
Čas: 14:46 31.03.2003
E-mail: motl@feynman.harvard.edu
Web: Kvarky a N=8 SUGRA
Pane Bedne, tak to se omlouvám, pokud jsem to napsal tak nesrozumitelně. Když tu šla řeč o elementárnosti kvarků, tak jsem naopak "odsoudil" výroky, že byla experimentálně nalezena vnitřní struktura. Všechny tyto výroky v minulosti se ukázaly být důsledkem chyby experimentátorů, případně statistické fluktuace, a proto nezbývá než zopakovat, že kvarky jsou podle všech známých experimentů bodové, elementární částice bez vnitřní struktury. V teorii strun je kvark přibližně řečeno jednou strunou s jistou formou vibrace, a tuto větu lze chápat jako popis "vnitřního uspořádání kvarků". Každopádně neexistují ani teoretické, ani experimentální důvody k víře, že je kvark složen z několika dalších, ještě elementárnějších částic.

K druhé otázce: N=8 supergravitace je čtyřrozměrná efektivní teorie pole, která obsahuje maximální možnou supersymetrii, jakož i obecnou teorii relativity. Do struktury teorie strun zapadá proto, že je dimenzionální redukcí 11-rozměrné supergravitace. 11 je maximální počet rozměrů časoprostoru, kde supersymetrická teorie existuje. N=8 supergravitace je proto dobrým nízkoenergetickým popisem M-teorie svinuté na sedmirozměrný torus (11-4=7).

N=8 supergravitace je jinak ovšem naprosto nerealistická teorie pro reálný vesmír. A vždycky byla, ačkoliv to někteří lidé neviděli okamžitě, a vzrušeni velkou mírou symetrie, snažili se teorii aplikovat na reálný svět. Vysoká míra supersymetrie (N=8 spinorů) má za důsledek nejen levo-pravou symetrii, odporující experimentům se slabou interakcí, ale i párování částic do početných rodin. Taková velká míra supersymetrie nejde ani pořádně narušit. Nerealistická je nakonec i N=4 a N=2 supersymetrie. Jediná realistická supersymetrie je N=1 supersymetrie ve čtyřech dimenzích. Ta předpovídá jednu novou částice - jednoho superpartnera - ke každé známé částici. Teorii s N=1 supersymetrií lze získat svinutím M-teorie na složitější variety, než je torus, konkrétně tzv. variety s holonomií G2.


Příspěvek od: Pavel Bednář
Čas: 13:29 31.03.2003
E-mail: zorl@seznam.cz
Pane Motl. Celkem dost krutě jste tady odsoudil elementárnost kvarků. Jaký je tady váš názor třeba na supergravitaci N=8? Vím, že nyní jsou i modernější teorie (p-brány) ,ale.....


Příspěvek od: Vojta Hala
Čas: 22:14 12.12.2002
E-mail: egg@matfyz.cz
Web: http://egg.matfyz.cz
OK, diky za objasneni. Clanek jsem nasel tak, ze na nej vedl odkaz z tohoto clanku na Neviditelnem psu, ktery
je take prekladem Cramera.
http://pes.internet.cz/veda/clanky/26882_0_0_0.html


Příspěvek od: Lubos Motl
Čas: 22:08 12.12.2002
E-mail: motl@feynman.harvard.edu
Web: Oprava
Zeptal jsem se Nimy Arkani-Hameda, coz je asi nejuznavanejsi casticovy fenomenolog dnesni planety (nebo jeden ze tri), a ten me ujistil, ze CDF opravdu oznamilo v roce 1996 nejaky prebytek nekolika udalosti s casticemi (srazky top-kvarku apod.), uplne na okraji spektra - tam jsou data, ktera nikdy nesmime brat moc vazne, protoze kdyby je mohli merit spolehlive, tak by jeste zkusili dalsi. ;-)

CDF se pokusilo vec promerit presneji a efekt proste zmizel. Cely pripad byl tedy nahodnou statistickou fluktuaci, ze ktere vubec nic neplyne. Nevim, z jakeho roku je Crameruv text, ale pokud je z poslednich dvou let, tak je to pro Cramera trochu ostuda. Pan Svrsek by asi take mel lepe vazit, koho prekladat.

Lisa Randallova, take jedna z nejslavnejsich fenomenologu, se shoduje, ze tyto vyroky "went away", ale rika neco podle me jeste pravdepodobnejsiho: ze proste pouzili spatnou strukturni funkci protonu pro male x, a pokud uzili lepsi, teorie se shodovala s pozorovanim.

Kazdopadne zadna takova anomalie neprezila, a proto nema smysl zadna otazka o tom, co takova anomalie znamena.


Příspěvek od: Lubos Motl
Čas: 21:57 12.12.2002
E-mail: motl@feynman.harvard.edu
Takze CDF objevil uz v roce 1996 preony haha. Tak to slysim opravdu poprve.

Vojto, asi bys mel cist takove veci kriticteji. Nejdrive si zjistit, proc kdo danou vec tvrdi a co za ni stoji, a pak se teprve ptat, jake ma dusledky pro zivot nebo treba pro teorii strun.

John G. Cramer - ktereho pan Svrsek prekladal - je tak napul autor science-fiction.

http://faculty.washington.edu/jcramer/

Vubec netusim, jak prisel k tomu, ze CDF tvrdilo, ze dokazalo vnitrni strukturu kvarku. Musel bych videt nejaky zdroj, zatim jsem videl jen nejake mlaceni slamy, a to jeste v prekladu.

Top kvark ma hmotu na urovni elektroslabeho sjednoceni, je tedy silne vazan na Higgse - proto fyzika z vyssich radu poruchoveho rozvoje je velmi dulezita a veci jsou znacne nelinearni - a slo o energie na hranici nasich znalosti. Exces eventu muze znamenat temer cokoliv, treba neprimy dukaz supersymetrie.

Tvrzeni, ze jde o dukaz vnitrni struktury kvarku, je dost silne.

Teorie superstrun je bohuzel dodnes dostatecne flexibilni na to, ze by mohla obsahnout i preony. Ale takhle otazka vubec nestoji. Podstruktura kvarku by nepochybne znamenala velkou zmenu i v nasem pohledu na popis castic pomoci efektivni teorie pole; nemusime chodit ke strunam. Je to nepochybne velmi odvazne vyjadreni uz proto, ze zadny fungujici model s preony - ktery by napriklad umoznil generovat 3 rodiny kvarku, ani vice, ani mene - zatim nikdo nepredlozil.

Kdybys dal odkaz na nejaky konkretnejsi pramen, mohla by byt konkretnejsi i moje odpoved. Z meho hlediska Ti zatim mohu jen rict, ze tim to pro me konci, protoze zatim takove vyroky povazuji za napul smyslene blaboly. Po pravde receno ani moc neverim tomu, ze CDF publikovalo kolem roku 1996 clanek, kde tvrdilo, ze nalezlo neprime dukazy substruktury kvarku.


Příspěvek od: Vojta Hála
Čas: 21:08 12.12.2002
E-mail: egg@matfyz.cz
Web: http://egg.matfyz.cz
Nektera experimentalni pozorovani nesmele naznacuji, ze kvarky maji urcitou vnitrni strukturu.

http://natura.eridan.cz/natura/2000/8/20000805.html

Co by to znamenalo pro teorii superstrun? Pokud se pamatuji, Brian Greene v Elegantnim vesmiru povazoval kvarky za elementarni, tedy za struny.


Příspěvek od: Jiří Haldovský
Čas: 19:58 02.12.2002
E-mail: monte_kristo@centrum.cz
Přeji dobrou chuť pokuď právě obědváte a protože s největší pravděpodobností neobědváte rád bych se vás zeptal : Jestli za předpokladu že existuje elementární částice která zajištuje interakci mezi objekty (třeba graviton nebo glukon). Existuje výraz který matematicky definuje vztahy mezi touto elementární částicí a objektem s kterým interaguje? Díky za případnou odpověď. S úctou,
mě k vám vlastní
Jiří Haldovský


Příspěvek od: Marek Pechal
Čas: 20:01 17.11.2002
E-mail: MPechal@seznam.cz
Vřelé díky za odpověď (to množství textu napsané za tak krátkou chvíli je obdivuhodné). Potřeboval jsem to vědět, abych si mohl odvodit vztah pro teplotu černé díry. Tak teď už to vím. A jsem spokojený. Určitě se tady někdy znovu objevím.


Příspěvek od: Lubos Motl
Čas: 22:43 01.11.2002
E-mail: motl@feynman.harvard.edu
Web: Jednotky
Mam jen dve minuty, a tak strucne. Teoreticti fyzici pracuji casto v jednotkach, kde je rychlost svetla a Planckova konstanta a Boltzmannova konstanta rovna cislu jedna. To znamena, ze mohou vzdalenost merit v sekundach (totez co svetelna sekunda), energii v inverznich sekundach a podobne.

Teplota a energie je jedno a totez. Teplotu muzeme merit v Joulech nebo elektronvoltech, pokud ji vynasobime Boltzmannovou konstantou (prepocet mezi udajem pro energii a udajem pro teplotu).

Formule S = A / 4G tedy, pri pouziti normalnich jednotek, obsahuje radu dalsich konstant, neco jako

S = k_{Boltzmann} . A / 4.G_{newton}.hbar_{Planck}.c

Nejsem si ted jist spravnou mocninou "c", ale to si jiste dopoctete sami. Kazdopadne neni absolutne nic spatneho na tom, pouzivat tyto jednotky, naopak je to symptom fyzikalni pokrocilosti. Lide zavedli ruzne jednotky pro cas a prostor jen proto, ze tehdy nemeli poneti o teorii relativity, a podobne pro energii a frekvenci maji jine jednotky proto, ze nevedeli nic o kvantove mechanice. A teplotu nemeri stejnymi jednotkami jako energii jen proto, ze nevi, ze teplota (vynasobena Boltzmannovou konstantou) predstavuje mnozstvi energie na dva stupne volnosti.


Příspěvek od: Marek Pechal
Čas: 22:33 01.11.2002
E-mail: MPechal@seznam.cz
Ahoj,
měl bych jeden dotaz:
B. Greene ve své knize Elegantní vesmír píše v 2. poznámce k 15. kapitole, že entropie černé díry je rovna ploše jejího horizontu dělené čtyřnásobkem Newtonovy konstanty. Jak můžu z tohohle vztahu dostat entropii, jejíž jednotka je J.K-1, když ve výchozím vztahu se nevyskytuje žádná konstanta, která by do "hry" přiváděla jednotku kelvin? Je možné, že je to nějaká tisková chyba, nebo existuje ještě nějaká jiný "druh" entropie?
Předem díky.
Marek Pechal


Příspěvek od: Michal Fabinger
Čas: 18:04 10.10.2002
E-mail: fabinger@stanford.edu
Treba na http://pancake.uchicago.edu/~carroll/notes/.

Zapsany strucne vypadaji takhle

R_{ab} - 1/2 R g_{ab} = -8 pi G_N T_{ab}.

Jinak v cestine existuje treba ucebnice obecne relativity od Kuchare, Zaklady obecne teorie relativity.




Příspěvek od: Petr Pernikář
Čas: 13:54 10.10.2002
E-mail: petrper@seznam.cz
Kde seženu všechny rovnice Einsteinovy obecné teorie relativity.

Za odpověď děkuji


Příspěvek od: Pavel Vesely
Čas: 14:05 08.10.2002
E-mail: vesely.p@post.cz
Web: www.exasoft.host.sk
Kde se daji sehnat vzorce pro pohyb planet, mesicu, komet...
ve slunecni soustave v zavislosti na case?

Jak se scitaji v relativite sily? Je to stejne jako u
rychlosti? A jak se odcitaji rychlosti a sily? (nebo staci
jen pricitat opacny vektor)

Jake jsou vzorce pro popis zakrivenych prostoru?
(hyperbolicky, sfericky)

Omlouvam se timto Vojtechu Halovi, ktery nelibe ponese, ze
se tu nediskutuje o strunach.

Pavel


Příspěvek od: Leoš Hromádka
Čas: 14:04 25.07.2002
E-mail: LHromadka@email.cz
Ahoj strunoví kosmologové

Dopadne na povrch černé díry vůbec něco?

Když se něco dostane do gravitačního pole černé díry pak je to k ní přitahováno. Stále to padá a pohyb se stále urychluje. Při vysoké rychlosti blížící se k rychlosti světla však dochází k přeměně padající hmoty na energii a ta je vyzařována do okolí. Část takového záření lze pozorovat a dnes se považuje za důkaz existence černé díry na daném místě.

Ale černá díra působí i na toto záření, jelikož jde konec konců o fotony, nebo ne? Jde nějak stanovit poměr záření, které je vyzářeno z dosahu černé díry a toho které je odsouzeno setkat se s horizontem událostí?

Neexistuje nějaká konstanta pro velikost (hmotnost) černé díry do které by nikdy nic nedopadlo, jelokož by se to při zrychlujícím se přibližování kompletně přeměnilo na energii a vyzářilo se?

Leoš


Příspěvek od: Leoš Hromádka
Čas: 13:46 25.07.2002
E-mail: LHromadka@email.cz
Ahoj strunoví kosmologové

Opravdu nic nemůže opustit černou díru (pominu-li záření absolutně černého tělesa)?

Ale co gravitace? Je-li gravitace zprostředkována gravitační silou předávanou gravitony, pak tyto procházejí přes horizont událostí, jako kdyby ani neexistoval. Je to tak, či je moje dedukce nějak špatná?

Je gravitační pole černé díry 100% homogenní? Nedalo by se z nerovnoměrností při přesném měření gravitační síly v okolí černé díry usuzovat něco o její vnitřní struktuře?

Leoš


Příspěvek od: Josef Nežerka
Čas: 12:21 18.03.2002
E-mail: josef.nezerka@email.cz
Muze mi nekdo poradit, kde se dovim neco vic o tachyonech nebo jsou uz tyto castice zcela zavrzeny? Diky


Příspěvek od: Vojtech Hala
Čas: 20:53 14.03.2002
E-mail: egg@matfyz.cz
Web: http:/egg.baf.cz
Mily pane Perno. Jestli Vase prispevky budou i nadale stejne smysluplne jako ty prvni dva, zacnu je mazat velmi brzy. Uzitek v nich zadny neshledavam.

Vsem hezky den!


Příspěvek od: tomas perna
Čas: 20:02 13.03.2002
E-mail: tomasperna@centrum.cz
Vazeny pane Steinere, nevim, co oznacujete pojmem "ubohe vylevy". Bylo by dobre nastvavat se presneji nebo, lepe, uplne presne. Co byste rekl kupr. tomu, kdyby teorie superstrun byla formulovana na fazovem prostoru takovem, ze vyhovuje funktoru ze symplekticke kategorie do kategorie Hilbertovy. Ja, protoze se zivim matematickym modelovanim, jsem neco na ten zpusob udelal a verte, prinasi to jen uzitek. Uzitek maji prinest i me "nasiraci" prispevky, ve kterych hodlam pokracovat, pokud mne nezacnou vymazavat. Zdravi Perna


Příspěvek od: Marcel Steiner
Čas: 15:59 13.03.2002
E-mail: marcel.steiner@volny.cz
Pane Perno,
Co je to za ubohé výlevy?!


Příspěvek od: tomas perna
Čas: 20:54 11.03.2002
E-mail: tomasperna@centrum.cz
Vazeni strunari ! Kladete nekdy zbytecne zasvecene otazky. Beztak Vam je nikdo dobre nezodpovi. Retezec stupnu abstrakce: castice-pole-grupa-struna sice Wittenovci vyuzivaji, ale vubec si s nim nevi rady. Furt ho rusi zavadenim mechanistickych predstav do vyssich symetrii, ktere maji akcni vlastnosti. Tim se to pak cele hybridizuje a zachvacuje jakoby dalsi rozmery. Vubec neni zrejme zakladni kladeni principu a logika teorie je neformulovatelna. Vypada to pak cele jako tlusta zenska s morem sminek a ovesena zlatymi superozdobami. -Domeny souradnych systemu nemohou najit samy sebe, natoz pak udalost a Jakobiany transformaci se bezmocne vazi na vazebnou konstantu, jejiz hodnota se tak vytraci. Zenska chudak se uz skoro nepohybuje, takze jeji svetoplocha je zavesena na svetelnem kuzeli jako kus hadru. Nahazime-li pak na tento hadr bodlaky, dostaneme vektorove svazy wittenovske K-teorie klasifikace D-bran. Kohomologie jak blazen, fakt. Servus ! Tom


Příspěvek od: Tomas Perna
Čas: 21:00 07.03.2002
E-mail: tomasperna@centrum.cz
Diskusni forum tohoto typu vitam. Je zde nekolik entit, ktere zasluhuji pozornost. Superstruny jsou zvlaste vybrana jsoucna: transformuji zrejme jako vektory, ale nejsou jimi. Nesou aspekty relativistickeho chovani, ale ztezi se mohou vazat na udalosti v 3+1, protoze kdyby se vazaly, byly by CY-variety celkem k ho.... S trochou matematiky mohu vykresat celou filosofii, kdyz udelam hybrid: struny v 3+1 plus jejich teorie na CY = likvidace horizontu udalosti. Pomuzu si cernou dirou na prave strane a spor na leve vyresim dualitami. Je to sranda. Hodne velka sranda je taky vzorec E=kT, pokud T neni alespon trochu spinova. Ale je-li trochu spinova, pak je jen trochu casticova. A je-li jen trochu casticova, pak ji lze tezko vazat na E. Je-li ovsem uplne spinova, pak je ji energie celkem vurt. Zajimalo by me taky, do jakeho pruseru by se dostal chudak Witten, kdyby se pokusil interpretovat vlnovou funkci ve vztahu k "logice" superstrun, pokud by si nepomoh T-ckem. Drzim palce a zdravim: Tom


Příspěvek od: Jakub Novák
Čas: 11:36 19.02.2002
E-mail: www.gongove@seznam.cz
Jeste k hmotnosti fotonu a o hmotnosti vubec

1. Newtonovo jablko
Kdyz panu Newtonovi spadlo na hlavu jablko, bylo tomu tak proto, že bylo hmotne. Z tohoto prikladu by se dalo usoudit, ze hmotnost je reakce s gravitaci. Gravitace reaguje s prostorem a zakrivuje jej a tak by se dalo rici, ze vse, co se pohybuje v zakrivenem prostoru je hmotne.
Kdyz posadime pana Newtona k cerne dire, muze mu na hlavu misto hmotne hmoty spadnout hmotna energie. Nikoli energie ktera se podle vzorce E=mc2 muze premenit na hmotu, ale energie, ktera je zaroven hmotou!
Proc?! Protoze pokud jsme si definovali hmotnost jako reakci s hmotnosti, pak muzeme pozorovat, jak foton pekne reaguje s gravitaci cerne diry a pada do ni.

2. Einsteinovo zrychleni

V jedne Einsteinove knizce jsem se docetl o hmotnosti jako o odporu pri zrychleni(pokud jsem to zpravne pochopil). Potom by mel foton nekonecnou hmotnost, nebot ma nekonecny odpor k zrychleni. Abychom mohli foton urychlit, musel by mit zápornou hmotnost nebo se zaporne pohybovat v case.

3. Zaver

Castice ve vesmiru se deli na bosony, tedy skoro energii a fermiony tedy skoro hmotu, pokud nepocitam cas prostor a gravitaci. Rozdil mezi homotou a energii by tedy byl v jejich spinu.
Zatim co energie, tedy treba foton nepotrebuje nic dalsiho ke svemu pohybu, hmota jako kvarky nutne k pohybu potrebuje energii.
Zatim co hmota ma ochocene sve vlastni gravitony, energie je pouze odkazana na hricky gravitonu okolni hmoty.

I rozdeleni hmoty a energie podle spinu vsak neni dokonale, nebot jako energii bereme i gravitaci, ktera vsak ma spin jiny.
Dalsim problemem jsou elektrony. Je to emnergie nebo ne?! Jsou hmotne, ale tvori elektrinu, kterou radime mezi energii.

Chyba je tedy v definicich a nazvech. Foton je hmotny, podle toho jak si definujeme hmotnost. Jasne je vsak jedno. At uz exsistuje v nasem vesmiru cokoli co je nehmotne, musi to alespon mit strukturu struny. I kdyz mozna jsou fotony nehmotne (tedy nemaji svou vlastni gravitaci?) nejsou rozhodne abstraktni, stejne jako neni nic abstraktni.

Jakub Novák


Příspěvek od: Jakub Novák
Čas: 11:10 19.02.2002
E-mail: www.gongove@seznam.cz
Mam pouze malou pripominku k odpovedi na dotaz jednoho navstevnika, jak zkonci nas vesmir. V teto odpovedi se tvrdi, ze takovato otazka nema smysl, protoze se neda experimentalne proverit.
S tim vsak nemohu souhlasit. Na tomto serveru probiha diskuse o teorii superstrun. Prave tato teorie nam o pulsovani vesmiru hodne rika. Diky jeji uprave relativity a kvantove mechaniky vime, ze se prostor nemuze zmensit v jakemkoli rozmeru na mensi vzdalenost nez na plankovu delku.
Dale vime, ze pri konecnem krachu naseho vesmiru se struny tvorici hmotu a energii zacnou naviet takovym zpusobem, ze se diky vlasnimu vybrovani odrazi od plankovy delky. To znamena, ze kdyz se nas vesmir dostane do velikosti bizke plankove delce, struny zvrati smrstovani a obrati jej na roztahovani.
Tak velky krach zkonci velkym treskem a pokracovanim noveho pulsu vesmiru!

Jakub Novak


Příspěvek od: Jaroslav Kolomazak
Čas: 11:01 05.02.2002
E-mail: kolomaz@karlin.mff.cuni.cz
Web: www.karlin.mff.cuni.cz
Vazeni pratele,
predne vam posilam mnoho nejsrdecnejsich pozdravu a drzim vam palce. Mam nekolik dotazu tykajicich se AdS/CFT.
1.Zajimala by me souvislost nedavneho Wittenova clanku o castecne nehmotnych polich v AdS/CFT korespondenci (hep-th/0109096) s obecnejsi teorii invariantnich diferencialnich operatoru v parabolickych geometriich na hranicich symetrickych prostoru, konkretneji se zobecnenymi BGG posloupnostmi. Jsem hluboce presvedcen, ze touto cestou se podari rigorozne formulovat tento zajimavy fyzikalni problem a prispet tak alespon stripkem k poznani teorie tohoto sveta.
2. Zajimala by me souvislost THC kvazineutralnich toku vznikajicich prirozenou redukci N=4 YM na hranici na sigma model na Riemanovske plose.

Jarda Kolomazak



Příspěvek od: Červený
Čas: 15:15 29.01.2002
E-mail: cerveny@kgb.cz
Na chování fotonu bych neviděl nic tak divného:
Právětak jako světlo prochází např. vodou prochází i vakuem.
Samozřejmě se nabízí otázka co se elektricky/magneticky ve vakuu polarizuje - zřejmě virtuální páry?!


Příspěvek od: Dan Cypris
Čas: 22:53 27.01.2002
E-mail: cyprisova@iol.cz
Dočetl jsem Elegantní vesmír s dost velkým skepticismem. Celé mě to totiž připomíná náboženství. Mám k tomu několik poznámek:

1. V knize se píše, že nemají smysl menší jednotky než Planckovy, ale nějak z toho vyplynulo, že superstruny se vlní menší vlnovou délkou než je Planckova.Jinak by při Planckově délce struny musela vyjít přesně 1 vlna na délku struny, při dvou 1 nebo 2 a tak dál. Podivné.
2. Vysloveně jsem nepochopil, proč jsou zmiňovány jenom Calabyho-Yauovy variety. Asi nejlíp zapadají do výpočtů a předpokladů ?
3. Uvažoval někdo, že vnímatelné rozměry nemusí být jenom prostorové a časové ale například energie částice může být dána nějakým stavovým rozměrem ?
4. Nechápu jak může struně chybět některý rozměr. Pokud je na papíře třeba jenom tečka, pak při uvažování dvou rozměrů musí mít délku a šířku a při třech rozměrech ještě i výšku. Jinak tam prostě není. Na singularity a bezrozměrné body nevěřím stejně jako na nekonečna.
5. Nepochopil jsem, jak se strunová teorie s pevně určenou strunou vyrovnává s kvantovou neurčitostí (třeba výskyt elektronu).
6. Pokud struna několikrát ovíjí svinutý prostor tak se namotává do svého vlastního těla? A když se vlní, neruší se ty vlny navzájem ?

Mám toho víc, ale zatím to stačí. Nemám tu správnou matematickou průpravu a jsem laik. A přestože jsem jakž takž strávil bez výhrad kvantovou teorii, toto se mi zdá spíš vroucným přáním fyziků, aby to tak opravdu bylo.


Příspěvek od: Háček
Čas: 22:00 26.01.2002
E-mail: Hook.soft@post.cz
Web: hook.czweb.org
Zdravím. jsem rozradostěn existencí této strany.
jsem laik a tudíž odpuste bude-li ma skromna otazka nebo spise zamysleni(u ktereho doufam v nejakou reakci od vas)ponekud krkolomna.

Foton je bezesporu nehmotny, dle Teorie rel., protoze kdyby nebyl mel by nekonecnou vahu, tedy vazil by jako cely vesmir a tedy foton by byl jen jeden a vyskytoval by se ve všech mistech vesmiru najednou(jak jinak). Avsak kdyz se kvantove mechanicky zamyslim nad svetelnou interferenci a pravdepodobnosti drahy fotonu, je pravdepodobnost drahy srovnatelna se vsemi myslitelnými dráhami(neni to dusledek vlneni?) a tudíž předcházejicí věta vede k zamyšlení.
A především proč má mít foton nulovou hmotnost, kdyz i u gravitonu se nejaka hmotnost ocekava. cozpak foton je pouze vlastnosti supercasoprostoru. ja si myslim, ze ano. jde o to proc vznika a vlni se zrovna tak jak se vlni.proc.proc?

Pozn.: Me vlneni neni fyz.vlnení.
Chybi mi terminologie a asi to pusobi ... , ja vim. proto doufam, ze se mi nekdo ozve


Příspěvek od: tomas
Čas: 13:39 21.01.2002
E-mail: lohes@centrum.cz
Máte tu docela zajímavé příspěvky, konkrétně ke superstrunám mám jednu připomínku. V kvaziálním vektoru kauzality dochází zejména vazbou zpětné interakce k deformaci achronologického jevu: vysvětlím-no prostě asymetrie superstrun se projevuje i ve globální implicitě a to zejména díky geniálnímu Lawerově vzorci Eˇ-nxˇ'kos2 ve vazbě na triangularitu.


s pozdravem DExtr


Příspěvek od: Havran
Čas: 15:57 10.01.2002
E-mail: havranp@prpf.cz
Pro L.Motla: Na svych strankach uvadite vyhrady P.Feynmana na teorii superstrun. Doslo jiz k takovemu pokroku, ze by byli nektere z nich vice ci mene vyvraceny?


Příspěvek od: Červený
Čas: 15:09 10.01.2002
E-mail: cerveny@nrg.cz
Pro p. Vraštila:

Vážený pane kolego, nechtěně jste se dotkl toho, co mě nejvíce irituje na současné fyzice. Když nějaký kreativista položí fyzikální rovnice v překvapivý sled, a je natolik schopný, aby je dotáhl k přijatelným řešením, zaměstná zbytek fyzikální populace tím, že se všichni "méně šťastní" snaží interpretovat a ujařmit stávající náhled na vesmír k obrazu těchto řešení a to s hrubým přehlížením experimentální zkušenosti a jiných souvislostí.

Proč se "strunovci" :-) nepokoušejí odvodit např. počet "nesvinutých" dimenzí, počet typů interakcí? Zamyslel se někdo např. nad tím, jak volně letící tělesa interaguje s prostorem (když to velmi zjednoduším jak poznají, že letí "rovně")?

Chtěl jsem tím pouze naznačit co v současné fyzice postrádám...

Zdraví Z.Č.


Příspěvek od: Zbynek Vrastil
Čas: 10:22 02.01.2002
E-mail: zbyna@psi.cz
Pro p. Cerveneho:

Pokud jde o tunelovani castic z cerne diry, tak to jsem cetl v chytre knize :-) Jestli si dobre vzpominam, byla to kniha Cerne diry a budoucnost vesmiru od Hawkinga, coz je (pokud jde o kvantove vyparovani cernych der) odbornik z nejpovolanejsich. Cerna dira je zvenku definovana svou hmotnosti, el. nabojem a momentem hybnosti. Kdyz se z ni vypari konkretni castice (treba elektron) tak se tyto vlastnosti zmeni tak, jako by cerna dira prislusny elektron ztratila. Z vnejsiho hlediska to tudiz muzeme chapat i tak, ze tam elektron nekde byl a protuneloval se ven. Stejne vsechny popisy tohoto jevu (mineno kvantove vyparovani) jsou pouze snahou nejak lidsky interpretovat vypocty, ktere jej popisuji.
Pokud jde o vasi intuitivni predstavu, ze by mela existovat zaporna gravitace, moje intuitivni predstava je zase ta, ze by exitovat nemela :-). Myslim, ze to, ze je gravitace vzdy pritazliva je jednou z fundamentalnich vlastnosti, na kterych stoji existence vesmiru. Vzdyt jedine tahle vlastnost (spolu s neomezenym dosahem) ji umoznuje, aby hybala vesmirem, prestoze je ze vsech interakci suverene nejslabsi. Kdyby naproti tomu elektromagneticka interakce nebyla pritazliva i odpudiva, gravitaci bychom mohli rovnou zrusit, protoze by si ani neskrtla. Podobne fundamentalni vlastnosti je taky treba odpudivost silne jaderne sily na velmi kratkych vzdalenostech, ktera zabranuje tomu, aby se jadro atomu spontanne nezhroutilo v cernou minidiru. Myslim si (ale je to ciste osobni esteticke hledisko), ze antigravitace by narusila tu krasnou vyvazenost vlastnosti jednotlivych interakci: gravitace jako jednoducha pritazliva sila, elektromagnetismus jako jednoducha "dvoupolova" sila, podstatne slozitejsi silna jaderna interakce, ridici se "barevnymi" naboji kvarku (o slabe jaderne toho zas tolik nevim).

S pozdravem, Zbynek Vrastil


Příspěvek od: Červený
Čas: 10:51 21.12.2001
E-mail: cerveny@kgb.cz
Pro p. Vraštila:

Vážený pane kolego, neodpustím si ještě několik poznámek ke gravitaci:

Intuitivně se vnitřek černé díry, tuším, chápe jako homogenní, totální nedostatečnost všeho, alepoň já si ho tak představuji. Vlatnosti černé díry, pokud existuje, jsou tak spíše opačnými vlastnosti okolí, kterému bylo něco odebráno. Kde by se pak braly částice, které by se mohly pokoušet protunelovat ven?
(:-) ... kdo kdy ale viděl černou díru, nebo mohl dokonce prostudovat její povrch, či nitro? Kdo kdy měřil gravitační chování antičástice? Kdo kdy detekoval gravitační vlny nebo měřil jejich rychlost? Může intentzita gravitace alespoň ve vlně záporná?... (-:)

Domnívám se, že dříve než je možné přikročit k úspěšnému kvantování gravitačního pole, měly by se tyto věci vyjasnit. Myslím, že úspěch kvantování elektromagnetického pole (vypočítání pozitronu, spinu..) byl již založen nekvantovým detailním popsáním chování elektromagnetického pole ve vakuu (něco se ve vakuu přece musí elektrostaticky [virtuální páry] a magneticky [spin virtuálních párů] polarizovat)! Podobný fundament pro gravitační pole chybí. Již se několikrát opakuji, ale nemohu se zbavit dojmu, že "vstup" záporné gravitace do problému by mohl znamenat průlom v kvantování gravitačního pole. Také nejsem žádný fyzik, ale myslím, že se v moderní fyzice se k neprospěchu věci preferují matematicko / estetické (:-)elegantní(-:) proudy, na úkor studia logiky vakua, interakcí, vývoje a podstaty vůbec. Místo efektních "nasvětlování" reality různými interpretacemi, postrádám pokusy o odvození počtu typů interakcí, dimenzí, modelů chování hmoty, důkazy sporem, "selský rozum"..

Dosti emocí a ještě praktickou poznámku k Vašemu názoru o gravitování fotonů: foton není "dobře" lokalizován, jeho "korpuskulární" projev se může objevit nahodile v kterém koliv místě? Kde by tedy měl gravitačně působit?

Zdraví a hezké svátky všem přeje Z.Č.


Příspěvek od: Zbynek Vrastil
Čas: 09:39 17.12.2001
E-mail: zbyna@psi.cz
Vazeny pane Cerveny,

diky za reakci, rad se pokusim vase namitky vyvratit :). Jsem presvedcen, ze fotony ke gravitaci prispivaji a to stejnym zpusobem, jako odpovidajici mnozstvi hmoty podle E=mc^2, protoze gravitace je vzajemne vymene gravitonu. Nejsem fyzik, a nikde jsem o tom primo necetl, ale verim ze je tomu tak a doufam, ze to nektery z fyzikalne povolanejsich prispevatelu potvrdi.
Pokud jde o vyparovani, ve svem (nepresnem ale nazornem) prikladu jsem pocital pouze s relativistickym pohledem na cernou diru bez zapocitani kvantovych efektu, ktere vyparovani zpusobuji. Pokud jde o tunelovani castic (nejen fotonu) tak jste uhodil hrebik na hlavicku :-). Jedno z vysvetleni kvantoveho vyparovani spociva prave v tom, ze se castice diky tunelovemu jevu dostane za horizont udalosti (ackoliv na to nema energii - o tom prave tunelovy jev je) a pokud ma dostatecnou rychlost, uz do cerne diry nespadne, nybrz leti pryc, jako by ji cerna dira vyzarila. Pravdepodobnost toho, ze se castice tunelovym jevem dostane "ven" je pochopitelne tim mensi, cim je cerna dira vetsi, ale vzdy je nenulova.
Rad bych upozornil, ze tento pristup k vyparovani cernych der je z fyzikalniho hlediska zcela ekvivalentni ke znamejsimu vysvetleni pomoci virtualnich castic. Jde jen o dva ruzne popisy tehoz jevu, ne o dve ruzne teorie.

Zbynek Vrastil


Příspěvek od: Červený
Čas: 11:07 12.12.2001
E-mail: cerveny@kgb.cz
Pro p. Vraštíla

Líbí se mi, že pracujete se zdravým "selským" ;-) rozumem, se kterým o problému přemýšlíte. Ale podle Vaší úvahy by se asi černá díra nemohla vypařovat vůbec (jak z ní vyvádět energii?) a to byste asi ranil řadu stoupenců Hawkingovy teorie vypařování černých děr. Mě se na Vaší úvaze nezdá to, že by částice s nulovou hmotou (fotony) měly
přispívat ke gravitační síle (to by snad potom musely interagovat s antihmotou?). Pokud ke gravicaci nepřispívají nabízí se jednoduché řešení:
Po snížení hmoty se zmenší kritický poloměr a fotony dobývající se ven utečou. V rukávu mám ovšem ještě "čínskou kartu" - tunelování fotonů z černé díry. To je však asi silný tabáček...

Zdraví Zdeněk Červený


Příspěvek od: Zbynek Vrastil
Čas: 20:53 10.12.2001
E-mail: zbyna@psi.cz
V souvislosti s anihilaci jsem si vzpomel na jednu vec, ktera mi pri cetbe Elegantniho vesmiru vrtala hlavou. V jedne kapitole (tusim, ze to byla kapitola o problemech poruchovych teorii) se v prikladu operuje s anihilaci elektronu a pozitronu. Dle knihy se srazi, anihiluji, vznikne foton. V poznamkach je uvedeno, ze jde o virtualni foton, ktery vzapeti vytvari novy par elektron-pozitron. Do te doby, jsem zil v predstave, ze energie (tedy fotony), ktera pri anihilaci vznika je zcela realna a ze tudiz, kdyz splacneme kus hmoty a antihmoty, vznikne spousta energie a poradny krater. Z toho, co jsem se docetl v EV jsem ziskal dojem, ze z fotonu hned zase vytori hmota a antihmota (ktera pripadne muze zase anihilovat, ale v prumeru a zdalky ze nic moc zajimaveho nedeje). Pokud by mi nekdo tenhle rozpor objasnil, budu mu vdecny.

Zbynek Vrastil


Příspěvek od: Zbyněk Vraštil
Čas: 15:29 06.12.2001
E-mail: zbyna@psi.cz
Zdravim. Prave jsem docetl poslednich x prispevku a nemuzu si odpustit jednu poznamku pro pana Cerveneho. Konkretne k jeho predstave, ze kdyz do "hmotove" cerne diry spadne kus antihmoty, tak by se mela jeji hmotnost snizit.
Zkusme si predstavit, co by se uvnitr takove cerne diry stalo. Ja vim, je to blbost, ale pro nazornost si predstavme, ze nekde uvnitr cerne diry sedi vsechny castice, ktere do ni spadly a nemuzou ven. Ted na tohle "jadro" cerne diry spadne zminovany kus antihmoty. Dojde k anihilaci, ktera vyrobi spoustu fotonu, jejichz celkova energie odpovida hmotnosti antihmoty a anihilovane casti hmoty. Ale ejhle - jsme pod horizontem udalosti, takze fotony nemuzou ven. Podle E=mc^2 je energie a hmota rovnocenna, celkova hmotnost cerne diry bude puvodni hmotnost+hmotnost antihmoty.
Cela tahle konstrukce je samozrejme fyzikalne nesmyslna, ale myslim, ze pomerne nazorne doklada, ze cernou diru nejake rozdily mezi hmotou a antihmotou nezajimaji.

S pozdravem,

Zbynek Vrastil


Příspěvek od: Honza
Čas: 10:22 03.12.2001
E-mail: Koutny
Web: janpok@post.cz
Ahoj strunari!
zajimalo by mne do jake miry lze ze strun "odvodit nas svet". Vzdy jsem se domnival, ze zakladnim postupem jak ze strun dostat to co vidime kolem sebe je kompaktifikace. Ale
v posledni dobe mne nekdo vysvetlil, ze je mozna i alternativa - tj. ze jsme vazani na brane ? Jak jednoznacne lze ze strun odvodit napr. standartni model ? Podarilo se ze strun. teorii ziskat alespon priblizny odhad napr. pro CKM matici? Dalsi moje hloupa otazka jest: nikde jsem nevidel odhad casu za jaky by elektron obihajici okolo jadra spadl v dosledku vyzarovani grav. vln ? Tusim , ze to bude pekelne dlouhy cas, ale neni i toto duvodem proc potrebujem kvantovou gravitaci ? A muj posledni dotaz se tyka strunovych korekci k Einsteinovym rovnicim - neni alespon nepatrna nadeje na jejich meritelnost?
Honza


Příspěvek od: Červený
Čas: 15:04 16.11.2001
E-mail: cerveny@kgb.cz
Pro p. Gabriela:

Přijal jsem Vaše doporučení, a přečetl jsem si zmíněný článek p. Chýly. Jakkoliv nesdílím nadšení pro superstruny,
a k některým vývodům strunové teorie mám silnou nedůvěru, neměl jsem z tohoto článku dobrý dojem. Použiji-li mírný výraz napadá mě "malost". Místo zásadních výhrad, podepřených shovívavým nadhedem, zde nalezneme ubohé pitvání - koneckonců nikoliv nepřirozených a nikoliv četných - výmětů zaníceného kolegy Motla "tupým" nožíkem...

Zdraví Z.Č.



Příspěvek od: Jan Gabriel
Čas: 14:21 11.11.2001
E-mail: gabriel.jan@post.cz
Web: http://www-hep.fzu.cz/Centrum/gr/greene.pdf
Hezký den.
Doporučuji všem čtenářům velice zajímavé Zamyšlení nad knihou Briana Greenea Elegantní vesmír a jejím překladatelem panem Lubošem Motlem, které napsal pan Jiří Chýla z Centra částicové fyziky,Fyzikální ústav AV ČR.
Vše najdete na adrese :
http://www-hep.fzu.cz/Centrum/gr/greene.pdf


Příspěvek od: František Lomoz
Čas: 21:30 07.11.2001
E-mail: flomoz@volny.cz
Dobré nerušené dny.
Po napsání mého posledního příspěvku mne napadlo jiné pojmenování Planckovy délky. Planckova délka je geometrickým průměrem poloviny Comptonovy vlnové délky a gravitačního poloměru téhož objektu.
V tomto spojení se pak potkává kvantová mechanika v podobě vlnové délky s gravitací reprezentované gravitačním poloměrem. Pane Motl mám pravdu?


Příspěvek od: Zdeněk Červený
Čas: 15:05 06.11.2001
E-mail: cerveny@kgb.cz
Ještě mi napadl jeden problém plynoucí z obecně přijímaného modelu vypařování černých děr. Zjednodušeně:
Máme nepatrnou černou díru odpovídající zkolabovanému elektronu (povšimněte si znaménka náboje "-"). Spadne do něj virtuální elektron a černá díra zanikne. Podle vás totiž odevzá svoji energii (2E) aby zahladila deficit způsobený vznikem virtuálního páru. Výsledek tedy je konečný zánik malé černé díry a odlétající pozitron. Opět si všimněte znaménka náboje "+". Neodporuje to zákonům zachování!?

Zdraví Z.Č.


Příspěvek od: Cerveny
Čas: 13:12 05.11.2001
E-mail: cerveny@kgb.cz
Vážený pane Luboši M.,

zřejmě moje námitky nebyly zcela dobře pochopeny (anihilace hmoty a antihmoty by asi neměla působit úsměvně), takže opět vstupuji do diskuse. Tedy zřetelněji:

Nedaří se mi přijmout Váš názor že:
1.
střetnutí hmoty s antičásticí její hmotnost hmoty sníží:
ale střetnutí černé díry (hmoty!?) s antičásticí její
hmotnost naopak zvýší (antičásticí v této úvaze zde pro jednoduchost nerozumím virtuální antičástici)
2.
Když má virtuální antičástice čas reagovat s černou dírou,
proč nemá čas reagovat s běžnou hmotou (proč vakuum svojí
antihmotnou složkou "neleptá" hmotu!?)
3.
Vznik každého virtuálního páru hmota/antihmota musí podle
Vás provázet dramatické vybuzení sice slabého, ale neomezeného gravitačního pole (podle Vás obě složky vytvářejí stejné gravitační pole/deformaci prostoru).
Oč "elegantnější ;-)" by bylo, kyby se gravitační
pole obou komponent v okamžiku vzniku rušila stejně jako např. pole elektrostatické!?
Průběžný vznik gravitačního potenciálu vznikajícího virtuálního páru by nenarůstal singulárním skokem ale spojitým vzrůstem, který by jaksi regulačně bránil přílišnému vzdálení obou komponent...

Prosím, nemusíte mi odpovídat, pokud tato diskuse přepíná Vaši trpělivost. Rozumím tomu, že je těžké tyto úvahy vyvracet: gravitační projevy antihmoty dosud nikdo nezměřil,
černé díry stále ještě pouze tušíme ("superstrunovci;-)" ovšem už vědí, že antidíry neexistují), velkému třesku nikdo neporozuměl (já ho např. chápu jako kondenzaci/krystalizaci vakuua), počet otevřených/uzavřených dimenzí vesmíru dosud nikdo neodvodil, počet elementárních sil také ne... Jako by to chtělo spíše nějakého "superDarwinistu".

Zdraví a tvůrčí invenci přeje Z.Č.



Příspěvek od: František Lomoz
Čas: 21:15 04.11.2001
E-mail: flomoz@volny.cz
Dobrý den
Mám skromný dotaz, zda strunová fyzika pracuje se zajímavým vztahem mezi Comptonovou vlnovou délkou částice, jejím gravitačním poloměrem a Planckovou délkou?
Tento jednoduchý vztah
Lambda * Rg/2 = Rp^2
Kde Lambda označuje Comptonovu vlnovou délku, Rg gravitační poloměr a Rp Planckovu délku. Součin Comtonovy vlnové délky s polovičním gravitačním poloměrem téže částice je vždy roven kvadrátu Planckovy délky, tedy konstantě.
Díky za odpověď.


Příspěvek od: Lubos Motl
Čas: 15:54 31.10.2001
E-mail: motl@feynman.harvard.edu
Web: http://www.kolej.mff.cuni.cz/~lumo/
Mily pane Kadleci,

zrcadlitou symetrii ve striktnim smyslu slova lze uplatnit jen na zvlastni tvary, konkretne Calabiho-Yauovy variety. Velke rozmery naseho vesmiru jsou tri, a tudiz nemohou tvorit Calabiho-Yauovu varietu, uz proto, ze ta musi mit sudou dimenzi. Proto obvyklou zrcadlitou symetrii nelze uplatnit na velke rozmery. Nicmene zrcadlita symetrie je blizkym pribuznym T-duality (viz kapitola 10 Elegantniho vesmiru o ekvivalenci malych a velkych polomeru). Podle T-duality se nas vesmir take da popsat jako vesmir o velikosti hypertrpaslicich 10^{-96} metru apod.

Pokud prejdete k T-dualnimu popisu, kde jsou polomery male, tak pohyb se zmeni na neco, co neni pohyb. Pohyb je popsan rychlosti - nebo hybnosti (pokud rychlost pronasobime hmotnosti). Ovsem hybnost objektu se pri provedeni T-duality zobrazi na neco jineho, konkretne na navijeci cislo (viz tabulky v kapitole 10). To, co vypada jako rychly pohyb kolem malinkeho vesmiru, je v nasem obvyklem jazyce jen nejaka rychla zmena kvantove faze daneho stavu, ktera se neda interpretovat jako pohyb v prostoru, a tudiz take nenarusuje zakon, ze rychlost svetla nelze prekonat.

Hezky den
Lubos


Příspěvek od: Lubos Motl
Čas: 15:47 31.10.2001
E-mail: motl@feynman.harvard.edu
Web: http://schwinger.harvard.edu/~motl/
Vazeny pane Cerveny,

omlouvam se za velmi dlouhou odmlku. Mel jsem plno jinych veci na reseni, ale konecne jsem si vzpomnel, ze jsem na Vas zapomnel. :-)

Cerna dira se vyparuje, ale pokud zaroven hlta hmotu, tak nejspise jeji hmotnost roste, protoze hltani je obvykle rychlejsi nez vyparovani. Kdyz se cpete tucnym obedem, tak Vase hmotnost take roste, ale to jeste neznamena, ze se pri tom nemuzete take potit. ;-)

Pokud v okoli cerne diry zadna hmota neni, tak je vyparovani jedinym procesem, a cerna dira proto ztraci hmotnost a energii.

Vase argumentace o tom, ze ma cerna dira vyparovanim nabobtnavat, je kulisacka; mohl byste byt dobry lichvar. ;-) Dobre vite, ze to tak nemuze byt. Na vytvoreni paru castice-anticastice je treba vynalozit energii "2E" ("E" na castici a dalsi "E" na anticastici). Tato energie se "uzme" z cerne diry a ta se stane lehci. Potom se jedna z paru vrati do cerne diry ("E"), ale druha vyletne ven, takze si celkove cerna dira ulehci o energii "E".

Vase poznamka o vytracejicim se zdravem rozumu mne pripada trochu usmevna: Vy nas chcete presvedcit, ze vyplivnutim castice hmotnost neztracite, ale naopak ziskavate, a naopak jedine vysvetleni teto otazky, ktere je v souladu jak s vedou, tak se zdravym rozumem, Vam pripada jako "vira" a "sterilni matematicka konstrukce".

Nuze, neni tomu tak. Kdyz se neco vyparuje, tak to ubyva, a nepotrebujete na takove tvrzeni slozite matematicke konstrukce z kvantove gravitace. Jinak vezte, ze jakmile do nejake otazky clovek vnikne poradne, tak mu exaktni zavery slozitych vedeckych konstrukci pripadaji zaroven i jako dilo "zdraveho rozumu".

Mejte se pekne
Lubos


Příspěvek od: František Lomoz
Čas: 21:54 26.10.2001
E-mail: flomoz@volny.cz
Dobrý den.
Předkládám něco k zamyšlení, k odmítnutí nebo něco zcela zcestného?
Strunová teorie nebo širší a hlubší M-teorie má ve svém vybavení rovněž časovou dimenzi. Na úrovni Planckových rozměrů hmoty, prostoru, momentu hybnosti a času má čas charakter nepřetržitosti nebo přetržitosti nebo snad nespojitosti?
Potom si mohu elementární částici představit jako troj-existenci jedné identity (struny), která se v přetržitém čase sbaluje a rozbaluje, takže výsledná uspořádanost např. elektronu je toho výsledkem. Pozorujeme tedy částici jejíž gravitační poloměr leží hluboko pod Planckovým rozměrem, avšak zároveň, elektromagnetické pole touto částicí reprezentované dává klasický poloměr elektronu, který vysoko převyšuje onen Planckův rozměr.
Osobně je mi přitažlivější představa o nulové střední hodnotě energie (hmotnosti) přetržitých vibrací oné troj-existence struny v čase, teprve hmotnost spojená s momentem hybnosti dává zrod hmotě elementární částice.


Příspěvek od: Zdeněk Červený
Čas: 14:55 15.10.2001
E-mail: cerveny@nrg.cz
Vážený pane Luboši,

Jsa panem Vojtěchem vytlačován z diskuse a Vámi topen v mléčku pomalu opouštím diskusi. Velké otazníky však v mém nazírání na svět prizmatem oficiální teorie zůstávají:

Např.:

Proč se vlasně černá díra vypařuje když hltá/stravuje jak
hmotu tak antihmotu?

Pokud antihmota nesnižuje hmotnost černé díry - její vypařování - co ji tedy způsobuje?!: Podle Vás tedy na úkor "zapůjčené" virtuální energie vznike pár hmota/antihmota, jedna z částic je nasána do čené díry, která tudíž !nabobtná! a zůstává lichá antičástice/částice a nevrácená virtuální energie? Z Vašeho výkladu jakoby černá díra neubývala ale naopak přibývala!? Snad by to "zachránily" nějaké fluktuace, výrony energie na povrchu černé díry nebo co, ale...

Vlastně jsem horší než ten nevěřící Tomáš. Po všech Vašich
vemlovaných slovech odcházím stejně, ne-li více rozpolcen.
Jistě to není Vaše vina, ale připadá mí, že se z fyziky
vytrácí "zdravý rozum a kauzalita" na úkor "víry" a sterilních matematickým konstrukcí, reprezantací, které mají
sytaktický půvab, vrhají stín nikoliv nepodobný realitě ale mající nulovou semantickou, vypovídací hodnotu.

S přáním, abyste si nenechal kazit náladu jedním skeptikem
se loučí Z.Č.



Příspěvek od: Vráťa Kadlec
Čas: 00:20 12.10.2001
E-mail: vratislavkadlec@centrum.cz
Web: kanal.ucw.cz
Dobry den, vy vsici, co tomu rozumite (nebo se tak tvarite),

Na uvad bych rad (ponekud alibisticky si vytvarim vymluvu k pripadnym blbostem psanym nize) zduraznil, ze krom jedne popularne vedecke knihy (hadejte jake) nejsem v oboru nijak vzdelan a i jinak mam k fysice velmi daleko. Ale k veci:

1) Je mozne uplatnit zrcadlitou symetrii i na tzv. velke rozmery? Zabyval se tim nekdo? Jestlize ne, proc, a jestlize ano, jaky tvar by mel parovy vesmir (mysleno ctyri velke rozmery) toho naseho (v jeho trech moznych tvarech - koule, sedlo a rovina {Me se zda nejhezci ten prvni, co vam?)?

2) Jak vypada vztah pod a nad plackovskych delek z hlediska rychlosti svetla? (Napr. predstavime-li si strunu navinutou na velke prostorove rozmery, pak by s jejiho pohledu bylo mozno prekonat vesmir stejne rychle jako {ted lovim v pameti, nevim zda jsem se trefil} 10 na -64 Planckovy delky.) Predpokladam, ze tento zdanlivy paradox lze v reci fysiku celkem jednouse odstranit, ale nejak si to nemohu predstavit..


Příspěvek od: Lubos Motl
Čas: 16:00 11.10.2001
E-mail: motl@feynman.harvard.edu
Web: http://www.physics.rutgers.edu/~motl/brian/
"Zachovani hmotnosti" v minulem textu melo byt "zachovani hybnosti". Vojto, trochu bych nesouhlasil, ze staci OTR. Na vyparovani cernych der samotna obecna relativita nestaci :-), musis pouzit i zakladni pravidla kvantove mechaniky a take pojmy termodynamiky. Fakticky vyparovani cernych der je prvni objev plynouci ze spojeni obecne relativity a kvantove mechaniky; v tom tkvi vyznam Hawkingova objevu. Hawking dokazal tyto uvahy a vypocty provest desitky let pred tim, nez byla dostatecne vypracovana teorie, ktera existenci vsech Hawkingovych jevu potvrzuje konkretnim mikroskopickym popisem (konkretne teorie strun).


Příspěvek od: Lubos Motl
Čas: 15:53 11.10.2001
E-mail: motl@feynman.harvard.edu
Web: http://www.matfyz.cz/lumo/
Mily pane Cerveny, je to legrace, jak porad nechcete verit tomu, ze se cerna dira vznikla z hmoty ci antihmoty absolutne nelisi, ale rad Vam odpovim i na Vase nove otazky. Na ty odpovedi byste jiste prisel sam, kdybyste se proste soustredil a zkusil trochu verit tomu, co rikam, ale jelikoz zatim nechcete, musim Vam do toho mlicka opet strcit cumacek.

Rozdelovani virtualnich paru je duvod vyparovani cernych der nejen laicky, ale v podstate presne takove jevy vypocet vysledneho Hawkingova zareni zahrnuje. Virtualni pary castice a anticastice v plochem prostoru vetsinou behem mziku zanihiluji. U horizontu cerne diry se vsak muze stat, ze jedna z castic vleti do cerne diry, druha vyletne ven (zachovani hmotnosti), a proto uz nikdy zanihilovat nemohou. Jedna z castic v paru tedy vyletne a bude se jevit vnejsimu pozorovateli jako vyparena castice, castice Hawkingova zareni.

Ovsem tato castice muze byt jak z "hmoty", tak z antihmoty! Vyparujici se cerna diry fakticky vysila stejne mnozstvi elektronu jako pozitronu. Stejne mnozstvi protonu jako antiprotonu! Kdyby tomu tak nebylo, tak byste mohl rozeznat cernou diru od anticerne diry (slozene z antihmoty), ale to opravdu nejde! Na tom vyparovani Vam musi byt skutecnost uplne jasna. Virtualni par castice a anticastice je neorientovany, a tak je pochopitelne stejna pravdepodobnost, ze vyletne jedna, nebo druha.

Cerna dira z antihmoty lze vytvorit, ale bude to uplne stejna cerna dira, jako ta z obycejne hmoty.

Zdravi
Lubos


Příspěvek od: Vojtech Hala
Čas: 15:22 11.10.2001
E-mail: egg@matfyz.cz
Web: http://www.matfyz.cz/egg
Dobry den vsem!

Nerad zasahuju do diskuse, ale zda se, ze to musi byt. Chtel bych pana Cerveneho poprosit, aby si otazky tykajici se cernych der apod. vyridil s Lubosem spise e-mailem, protoze na jejich zodpovezeni v podstate staci OTR a maji pramalo spolecneho s teorii superstrun. Luma bych chtel zase poprosit, aby napriste spolknul svoje poznamky o nekvalitnich operacnich systemech apod., protoze rovnez nemaji nic spolecneho se strunami. Co je kvalitni a co nekvalitni je otazka nazoru, kazdopadne diskusni tema tohoto fora je jine, tak se ho drzme.

Mejte se pekne, Vojta


Příspěvek od: Zdeněk Červený
Čas: 10:18 11.10.2001
E-mail: cerveny@kgb.cz
Vážený pame Luboši,

protože jste neodolal a nechal se vtáhnout zpět do diskuse
hraničící s půtkařením, dovoluji si Vás zeptat na názor
na následující otázku:

Myslím (nejsem si tím ovšem jistý), že pády virtuálních antičástic do černé díry je vysvětlováno její vypařování. (Tato teorie ovšem mi vyloženě nesedí: proč do černé díry nepadají také virtuální částice?) Otázka tedy v tomto kontextu zní: Jak taková antičástice tedy pozná, že spadla
do černé díry a ne do antidíry? (která podle mého jitřícího nározu nelze setrojit)

Zdraví Zdeněk Červený


Příspěvek od: Lubos Motl
Čas: 21:06 10.10.2001
E-mail: motl@feynman.harvard.edu
Web: http://www.matfyz.cz/lumo/
Vazeny pane Cerveny,

v nasem vesmiru je hmotnost hmoty asi tak pet vodikovych atomu na metr krychlovy. Podle mych meritek to znamena, ze nam toho moc nezbylo. To proto, ze kdysi davno temer veskera hmota zanihilovala s antihmotou; prevaha hmoty nad antihmotou byla jen "lehka". Kdyby byla prevaha hmoty ne jedna miliardtina (miliontina promile), ale "tezka prevaha", mozna bychom dneska meli slunecni soustavu a Galaxii precpanou. :-)

Pane Cerveny, cerne diry vznikle z antihmoty se opravdu vubec, ale vubec neodlisuji od cernych der vzniklych z hmoty obycejne. To mimochodem take znamena, ze spolu takove dve cerne diry v zadnem pripade neanihiluji. Jsou to obycejne dve cerne diry, stejne obycejne dve cerne diry, jako jakekoliv jine dve cerne diry. A dve cerne diry, pokud se setkaji, se vzdycky postupne spoji do jedne, vetsi cerne diry. Nikdy neanihiluji. Cerne diry se jen pomalu vyparuji, a jakmile se spoji do vetsi cerne diry, tato nova velka cerna dira se vyparuje jeste mnohem pomaleji, protoze vetsi cerne diry maji mensi teplotu. Kazdopadne neexistuje zpusob, jak cernou diru zlikvidovat rychleji, nez to dokaze vyparovani.

Zanihilovat muze elektron s pozitronem nebo proton s antiprotonem. Ale zadne cerne antidiry neexistuji; vsechny cerne diry jsou stejne. Cerne diry jsou v tomto smyslu "neutralni", maji stejne daleko od hmoty jako od antihmoty. Tyhle veci vime naprosto jiste, a pokud by na nich bylo neco zakladniho spatne, potom bychom se museli vzdat v podstate uplne vseho, co o cernych dirach vime.

Hezky den preje
Lubos


Příspěvek od: Zdeněk Červený
Čas: 13:37 10.10.2001
E-mail: cerveny@nrg.cz
Vážený pane Motle,

také nic ve zlém, ale žádnou odpověď již po Vás nepožaduji.

Nicméně si nemohu odpustit několik poznámek na závěr:

- pokud jsme svědky "lehké" převahy částic nad antičásticemi, neumím si představit "těžkou"...

- pokud se hmotové a antihmotové černé díry ničím neodlišují, jak mají poznat, že při setkání mají anihilovat?

- ani mi nedalo takovou práci najít na internetu, že nejsem
sám, kdo na má, na rozdíl od skalních "biliardtinářů";-),
jisté pochybnosti:

http://www.laeff.esa.es/eng/papers/abs94/laeff-9414.html:
(...Any ideas must confront the present agreement with general relativity, but yet be free to wonder about not understood phenomena, such as the dark matter problem and the anomalous spacecraft data which we announce here. This all has led some "intrepid" theorists to consider a new gravitational regime, that of antimatter. Even more "daring" experimentalists are attempting, or considering attempting, the measurement of the gravitational force on antimatter, including low-energy antiprotons and, perhaps most enticing, antihydrogen.)

s úctou a přátelským pozdravem Zdeněk Červený


Příspěvek od: Lubos Motl
Čas: 15:37 09.10.2001
E-mail: motl@feynman.harvard.edu
Web: http://www.kolej.mff.cuni.cz/~lmotm275/
Pane Cerveny, nic ve zlem, ale jestli nejste potizista, tak bych radeji nechtel videt, jak vypadaji potiziste. ;-)

Odpovedi temer na uplne vsechny otazky, ktere opakujete, uz padly, ale asi jste se rozhodl jim neverit. Pane Cerveny, jeste jednou, ale ode me naposledy.

Neni pravda, ze se antihmota odpuzuje. Antihmota se pritahuje a ma kladnou hodnotu gravitacni hmotnosti - coz, jak vime, je totez jako setrvacna hmotnost; tvrzeni, ze se tyto hmotnosti vzdycky rovnaji, je obsazeno v principu ekvivalence, ktery je experimentalne take overen s presnosti na biliardtiny.

Kladnost energie anticastic je naprosto spolehlive overena experimentalne, a dokonce i kdybyste si predstavil, ze overena neni, z teoretickeho hlediska neexistuje zadna moznost, jak popsat antihmotu se zapornou hmotnosti, aby vysledna teorie byt jen vzdalene pripominala nas svet.

"Rozsahle teorie" o antihmote, konkretne standardni model, ktery krome antihmoty popisuje i uplne vsechno ostatni, co kdy kdo pozoroval (vyjma gravitace), byly spolehlive overeny; v mnohych pripadech se teorie shoduje s experimentem na 10 desetinnych cislic i vice. Vysledek experimentu, ktery je casto nejasny pro laika, je casto naprosto jasny a trivialni pro fyzika - a naopak.

Kosmologickou konstantu nelze pocitat jako uhrnnou hmotnost nejake kase. Vypocet kosmologicke konstanty je velmi konkretni a specificky vypocet odvozeny z kvantovych teorii pole nebo jejich zobecneni - a muzete si predstavit, ze kladna hustota vakua pochazi z kladnych energii zakladniho stavu harmonickych oscilatoru, odpovidajicich bosonum (casticim s celym spinem), v kvantove mechanice. Zaporny prispevek ke kosmologicke konstante maji fermiony, castice splnujici Pauliho princip (tedy s polociselnym spinem). Znamenko velikosti kosmologicke konstanty neurcuje otazka, zda je castice castici, nebo anticastici (to je nakonec jen konvence - souvisejici s tim, ktere ze dvou opacnych castic je vice ve svete kolem nas -, napriklad nema smysl rikat, ze vyrok "W+ boson je castice a W- anticastice" je lepsi nez vyrok opacny). Znamenko prispevku ke kosmologicke konstante je dano tim, jestli je castice boson, nebo fermion, nikoliv tim, jestli je "castice" nebo "anticastice".

Cerne diry vznikle z hmoty se absolutne neodlisuji od cernych der vzniklych z antihmoty: vsechny jejich makroskopicke vlastnosti, naboje a ostatni "klasicke" veliciny jsou naprosto totozne. Charakter cerne diry v dane teorii urcuje jen jeji hmotnost, presne se zachovavajici naboje (v nasem svete pouze elektricky; pri vzniku a vyparovani cernych der se nezachovava ani baryonove cislo) a moment hybnosti. Nicim jinym se cerne diry nemohou odlisovat; znamy teorem rika, ze cerne diry "nemaji vlasy".

Lehka prevaha hmoty nad antihmotou se vysvetlovala narusenim CP symetrie, ale objevily se pochyby, jestli je toto naruseni dostatecne; jestli je treba se odkazovat na fluktuace nebo vubec nahodu apod. Kazdopadne neexistuje jasny doklad pro nazor, ze je neco s nasimi teoriemi v tomto ohledu urcite v neporadku.


Příspěvek od: Zdeněk Červený
Čas: 09:50 09.10.2001
E-mail: cerveny@kgb.cz
Nezlobte se, možná jsem úplně mimo, ale domníval sjem se, že pokud se hmota gravitačně přitahuje a antihmota (případně)odpuzuje je vakum (kaše virtuálních párů) gravitačně inertní. To jsem měl na mysli tou kosmologickou konstantou..

Jinak se jako laik trošku pozastavuji nad tvorbou rozsáhlých teorií o antihmotě aniž někto ověřil její vlastnosti. Nepochybuji o kladné "dynamické" hmotě antičástic, ale nejsem si jist kladnou "gravitační" hmotou...

Mimochodem jak (jakým kvantovým číslem) by se od sebe odlišily černé díry vytvořené hmotou a antihmotou?
Nebo z jiného soudku: nemáte problém s vysvětlováním nadbytku hmoty nad antihmotou?

Ještě jednou se omlouvám, nejsem žádný potížista, nemusíte mi odpovídat, ale komu jinému bych se měl se svými pochybnostmi svěřit...



Příspěvek od: Lubos Motl
Čas: 15:24 08.10.2001
E-mail: motl@feynman.harvard.edu
Web: http://www.kolej.mff.cuni.cz/~lmotm275/
Antihmoty prilis vyrobeno nebylo, takze se toho moc s gravitaci antihmoty primo experimentalne nezkouselo. Kazdopadne je mimo jakoukoliv pochybnost experimentalne i teoreticky dokazano, ze antihmota se sklada z castic, ktere maji kladnou energii, uplne stejne jako jakekoliv jine castice (otazka, co je hmota a co je antihmota, je castecne konvence) a diky vztahu E=mc^2 maji anticastice i kladnou hmotnost. Hmotnost udava, jak silne gravitacni pole hmota vytvari, a proto veskera hmota vytvari pritazlive gravitacni pole.

Naopak vsechna telesa zrychluji v gravitacnim poli stejne. To je zaklad obecne relativity. Vsechny deje ve volne padajicim vytahu museji probihat stejne jako ve stavu beztize, coz znamena, ze i antihmota pada smerem dolu v gravitacnim poli.

Kdyby ve vesmiru mohly existovat castice se zapornou hmotnosti (napriklad tachyony), zpusobilo by to naprostou nestabilitu vesmiru, protoze by dvojice castic s pozitivni resp. negativni energii mohly vznikat samovolne z vakua, aniz by to narusilo jakykoliv zakon. To rozhodne nenastava.

Obavam se, ze s kosmologickou konstantou tenhle problem vubec nesouvisi, a kdyby to fungovalo podle Vas, tak bude vesmir v cudu behem zlomku sekundy.


Příspěvek od: Zdenek Červený
Čas: 11:07 08.10.2001
E-mail: cerveny@kgb.cz
"Gravituuje" antihmota?

Dovolte mi prosím zeptat se někoho sečtelejšího:
Bylo vůbec ověřeno, že antihmota se navzájem gravitačně přitahuje?

Promiňte mi subjektivní názor, ale nebylo by přirozenější
kdyby se antihmota gravitačně odpuzovala a hmota s antihmotou byly gravitačně inertní? Myslím by to mohlo
snadno vysvětlit nulovou kosmologickou konstantu...

Děkuji za odpověď


Příspěvek od: Lubos Motl
Čas: 16:59 29.09.2001
E-mail: motl@feynman.harvard.edu
Web: http://www.kolej.mff.cuni.cz/~lmotm275/
Ahoj Tomasi! Prave jsem stravil hodinu psanim odpovedi na Tve dotazy. Netscape v Linuxu najednou naprosto bezduvodne zacal tuhnout, posouval jedno pismeno za 10 sekund a vrzal pri tom silene hard diskem. Asi po 15 minutach cekani na to, az se vzpamatuje, Linux zkolaboval uplne a restartoval se, aniz by zanechal cokoliv z rozpracovane odpovedi. Osobne bych tento nekvalitni operacni system nenainstaloval ani na pocitace na chude univerzite v Ukrajine, natoz sem. Ale presto zkusim napsat jeste jednou alespon podstatne body me predchozi odpovedi.

Obavam se, ze Te neuspokojim vice nez Tva ucitelka. Vesmir se rozpina, ale opravdu se nerozpina v nicem vetsim, protoze nic vetsiho neexistuje. Predstav si, ze nafukujes gumovy balonek. Rozpina se a rozpina se uvnitr naseho trojrozmerneho prostoru. Ted si ale predstav dvourozmerne cerviky, kteri jsou uvezneni do vnitrku gumy. Deni mimo plochu balonku je navic vubec neovlivnuje. Z hlediska jejich fyziky vnejsek vesmiru neexistuje. Muzes sice namitat, ze ackoliv cervici vnejsi prostor nevidi, stale existuje. Ale to plati jen proto, ze je analogie s balonkem nedokonala. Realny vesmir je synonymum pro vsechen prostor, ktery vubec existuje, a nic mimo nej neni. Priroda se obejde bez berlicek, jako je treba pomyslne prostredi, do ktereho je treba vesmir ponorit.

(Znalci modernich scenaru branovych svetu ted mohou namitnout, ze nas vesmir muze byt membrana vznasejici se ve svete s vice rozmery. Ale ja chci mluvit o celem svete, vcetne rozmeru kolmych k membrane.)

Vesmir, jak si ho predstavujeme, nema zadne hranice, ale presto muze mit konecny objem - podobne, jako ma balonek konecny povrch. Kousky balonu lze slepit z platku gumy, a stejne tak vesmir lze "sesit" z trojrozmernych "caru" prostoru: v jazyce matematiky, slepenim ruznych "map" z "atlasu" ziskame "varietu", coz je oznaceni pro prostor, ktery muze byt i konecny a muze obsahovat ruzne "cervi diry" a analogicke spojnice mezi ruznymi oblastmi. Kazdy jednotlivy kousek vesmiru vypada stejne jako kus plocheho prostoru a neni na nem nic spatneho. Jsou slepeny stejne, jako se lepi kousky prostoru, chceme-li vytvorit nekonecny trojrozmerny prostor. A proto musime pripustit, ze je mozne, ze je vesmir konecny, ale nema hranice. Proste si musime predstavit, ze kdyz poletime stale jednim smerem, muzeme se nakonec vratit na stejne misto, odkud jsme vystartovali. Je to podobne, jako kdyz nekdo obepluje zemekouli, ale v pripade vesmiru se obejdeme (a v podstate se musime obejit) bez dalsiho prostoru "mimo" vesmir, analogickeho vnitrku zemekoule nebo vnejsimu prostoru. Zakriveny vesmir, ktery je vnoreny do plocheho prostoru s vice rozmery, je prilis detska predstava. Opravdovi fyzici pracuji s uplne obycejnym trojrozmernym vesmirem (navic s jednim casovym rozmerem), kteremu umozni se zakrivovat (podobne jako mapa sveta zachycuje geometrii povrchu Zeme zkreslene) a ruzne napojovat jednu oblast na druhou (jako kdyz reknes, ze Aljaska uplne nalevo na mape je vlastne jen kousek od Kamcatky nebo Cukotky, ktere jsou zakresleny vpravo).

Co bylo pred velkym treskem? Ono opravdu nic nebylo. Ona vlastne slovni vazba "pred velkym treskem" ani nedava smysl, jsme-li dostatecne prisni. Je to neco podobneho jako "blize stredu Zeme, nez je stred Zeme" nebo jeste lepe "na sever od severniho polu". Severni pol je nejsevernejsi bod zemekoule a nic od neho na sever neexistuje. Stejne tak velky tresk je prvni okamzik naseho vesmiru a nic "pred nim" neexistuje. Samotny pojem casu prestava mit smysl, pokud se dostanes az k samotnemu velkemu tresku - podobne, jako svetove strany nemaji obvykly smysl, pokud dojdes az k severnimu polu.

A ze v Tobe vyvolava nic depresivni pocity nebo Te dokonce desi? To je sice hezke, presneji receno to nam sice muze byt lito, ale je to asi to nejmensi. Depresim muzeme propadnout, pokud se podivame na hrbitov, kde jsou pochovany obeti nejake tragedie. Nebo kdyz musime znovat psat desitky kilobajtu proto, ze na zdejsich pocitacich bezi nekvalitni operacni system. Ale kdyz pujdeme o 60 let zpatky a podivame se, co delali Nemci v koncentracnich taborech, deprese nestaci. Zacneme zvracet. Kdyz se vratime o par tisic let zpatky, prestane existovat jakakoliv lidska kultura podobna te dnesni. Tri miliony let v minulosti uz nenajdeme ani zadne lidske bytosti. Kdyz se vratime o par miliard let, zmizi jakykoliv zivot, a pak i nase planeta, Slunce, ostatni hvezdy. To uz jsme dost daleko od pece v Horni Dolni, kde travime vetsinu zivota, na ktery jsme zvykly a ktery formuje nasi predstavivost. Jak se priblizujeme k velkemu tresku jeste vice, zjistime, ze uz neexistuji ani zadne atomy (a skrz vesmir vubec neni videt) a pak prestanou existovat i protony a neutrony a vsechno vypada jinak. Zacinaji ztracet smysl nase obvykle pojmy trojrozmerne geometrie a v okamziku velkeho tresku zmizi i pojem casu samotneho.

Porad si to nedokazes predstavit? Nejsi sam. Priroda si zvolila pro prvni okamziky sveho dila velmi extremni podminky, ktere si opravdu zive nedokaze predstavit nikdo. Priroda by to samozrejme rekla uplne jinak: zvolila si celkem normalni podminky a zakony, ale prdolove, kteri par let - z hlediska vesmiru zanedbatelnou dobu - travi par kilometru od nejakeho mestecka, proste nemaji dost rozhledu a fantazie, aby si vubec dokazali predstavit, co priroda musela udelat. Priroda dala novemu vesmiru do vinku extremni podminky, vuci nimz vyhlizi vsechny duvody nasich radosti i depresi jako nepodstatne epizodky. Priroda ma plne pravo se pohrdave ofrnovat nad nasimi pocity. Pro vedce je ale dulezite to, ze dokazou vytvaret na zaklade svych teorii experimentalne overitelne predpovedi - a ze tyto predpovedi souhlasi s tim, co pozorujeme, a ze tedy vsechno do sebe zapada. Vedec musi byt neustale pripraven rozsirovat svou predstavivost a zbavovat se predsudku. Kdyz se podivas na sve pocity nezaujate, budes si muset priznat, ze nemas vubec zadny argument, proc by "pred velkym treskem" neco melo byt. Vzpomen si, ze lide v historii meli hromadu predsudku, ktere se ukazaly byt naprosto chybne. Zeme byla placka uprostred vesmiru, kolem niz obihala vsechna kosmicka telesa. Clovek se zasadnim zpusobem lisil od ostatnich zivocisnych druhu, ktere spolu navzajem vubec nesouvisely a ktere byly stvoreny jednotlive. Cas ubihal pro vsechny stejne, od minus nekonecna do nekonecna, a vesmir mel presne euklidovskou trojrozmernou geometrii. Objekty mely presne polohy a rychlosti. Vida, vsechno se ukazalo byt spatne. Vedci opravdu nemohou stavet na svych pocitech a emocich - presneji receno predsudcich - ale na korektnich vedeckych argumentech. A zvu Te, abys to take zkusil.


Příspěvek od: Tomáš
Čas: 01:46 29.09.2001
E-mail: NTomas@atlas.cz
Ahoj všem.
Chtěl jsem se zeptat někoho kdo tomu trochu víc rozumí než moji učitelé. Říká se, že se vesmír rozpíná ale v čem se rozpíná ? Dále mě trápí i něco jiného. Co bylo před velkým třeskem. Naše učitelka říká, že nebylo nic. Ale slovo nic ve mě vyvolává velkou prázdnotu. Nedovedu si totiž představit nic. Nic je něco tak depresivního. I prostor je něco ale nic si prostě nedovedu představit. Nic mě neděsí víc než nic. Dovedu si představit, že jednoho dne nebudu. Ale nedovedu si představit, že nebylo nic. A pak náhle z ničeho něco vzniklo to je prostě nepochopitelné. Budu velmi rád za vyčerpávající odpovědi.

Ahoj Tomáš


Příspěvek od: Vladimir Busek
Čas: 16:35 24.09.2001
E-mail: busek@post.cz
Dekuji za odpoved.

Ono to mozna z meho predchoziho prispevku nebylo prilis znat, ale ja uznavam, ze to zcela nechapu. Jako dalsi miliarda lidi si intuitivne myslim, ze zdroj vsesmerovy zdroj paprsku umisteny v letadle, ktere se pohybuje rychlosti blizke rychlosti svetla, bude emitovat svetlo, ktere bude v jednom smeru stat (skoro) a ve druhem bude mit rychlost jdouci k 2*c :-) (pro externiho pozorovatele). A protoze vim, ze to tak neni (rekli mi to na prednaskach a pisou to v knihach), snazim se to pochopit. Vetsinou je to osem psano jako axiom, o kterem se nediskutuje.

Jdu si tedy precist neco o Lorentzovych transformacich a jestli nekoho ma nevedomost urazi (rozesmiva, prekvapuje), tak s tim holt nic neudela, maximalne mi to muze vysvetlit, popripade dat odkaz na nejakou litereaturu/web.


Příspěvek od: Lubos Motl
Čas: 16:36 20.09.2001
E-mail: motl@feynman.harvard.edu
Web: http://www.matfyz.cz/lumo/
Mily pane Busku,

to vite, Vase otazka miri do podstaty teorie relativity. V podstate jinymi slovy rikate, ze jste v zivote o teorii relativity neslysel.

Konstantnost sireni rychlosti svetla je jeden ze dvou postulatu specialni teorie relativity. Druhym postulatem je, ze zakony plati stejne pro vsechny pozorovatele, kteri se vuci sobe pohybuji rovnomerne primocare.

Co se tyce konstantnosti rychlosti svetla: ano, mate pravdu, tento fakt protireci nasim zkusenostem s pohybem vlaku apod. Nicmene je pravdivy. Byl spolehlive overen experimentalne, a to dokonce jiz v predminulem stoleti. ;-) Na konci 19.stoleti provedli Michelson a Morley radu mereni, jejichz ukolem bylo namerit rychlost svetla z hvezd vuci Zemi. Pamatujte, ze Zeme se pohybuje sem a tam vuci Slunci (coz je lepsi vztazna soustava), a tudiz bychom se meli pohybovat vstric svetlu a zpet. Meli bychom v prubehu roku namerit ruzne rychlosti - pozorovat tzv. "eterovy vitr". Ale to se nestane. Experiment ukazal naprosto jednoznacne, ze je rychlost svetla vzdycky stejna.

Od Einsteinova objevu v roce 1905, kdy Albert dokazal z techto znamych faktu vydedukovat vsechny zazracne predpovedi specialni relativity, uz nas deli skoro stoleti. Pro fyziky, kteri alespon trochu pracuji s moderni fyzikou, jsou samozrejme vsechny tyto veci samozrejmostmi, vsem rozumime a denne tajemne predpovedi relativity vidime v miliardach pokusu na urychlovacich i jinde. Kazdopadne pro lidi mimo fyziku je to porad "nova vec". Pro rychlosti srovnatelne s rychlosti vlaku jsou nove efekty specialni relativity nepodstatne. Proto nejste sam, kdo se podivuje, ze rychlost svetla se nesklada scitanim - urcite existuji miliardy lidi, kteri v zivote o nicem podobnem neslyseli. :-)

No vidite, a presto je rychlost svetla konstantni. Muzete nam, fyzikum, neverit, ale bylo by to - jemne receno - neobycejne stupidni. :-)

Jeste komentar k Vasim argumentum. Nechapu, jak z vazby "fyzicka podstata" ma plynout, ze se vsechny rychlosti maji skladat scitanim. Vezmete si na pomoc autoritu "fyzicke podstaty" a s jeji pomoci pak obhajujete sve nespravne predsudky o fungovani sveta? To prece ne. Spravne skladani rychlosti u,v je nasledujici. Pokud se A priblizuje zleva rychlosti u a B se priblizuje rychlosti v zprava, potom vzajemna rychlost je rovna

(u+v) / (1+u.v/c^2)

Vsimnete si, ze kdyz jsou u,v male proti rychlosti svetla, jmenovatel je skoro jedna, a proto je vzajemna rychlost temer rovna u+v. Pokud je ovsem jedna z rychlosti rovna c, napriklad u=c, potom dostaneme (c+v)/(1+v/c) = c (staci vykratit). Tedy vzajemna rychlost svetla vuci cemukoliv je vzdycky rovna "c". Protoze je tato rychlost pro nas svet naprosto fundamentalni, fyzici uz pres 20 let uzivaji definici metru, podle ktere je rychlost svetla (ale i sireni gravitacnich impulsu, nebo nejvyssi mozna rychlost sireni cehokoliv) rovna presne 299 792 458 m/s.

Predstavy, ze je vzajemna rychlost vzdycky u+v, je proste dusledkem nespravne intuice, ktera se vyvinula proto, ze zijeme ve svete velmi malych rychlosti, kde u+v plati s ohromnou presnosti.

Mimochodem kdyz budete cist Elegantni vesmir, ceka Vas jeste asi desetinasobek mnohdy jeste hure stravitelnejsich tvrzeni. A presto, jsou pravdiva.

Zdravi
Lubos


Příspěvek od: Vladimir Busek
Čas: 08:50 20.09.2001
E-mail: busek@post.cz
Predem rikam ze nejsem fyzik :-) tak aby nedoslo k omylum.

Koupil jsem si Greenuv "Elegantni vesmir" (diky Lumovi za preklad) a hned ze zacatku mne tam neco zarazilo. Ac jsem vystudoval VS a zkousky z Fyziky mam, tak jsem si uvedomil, ze mi neni jasne, proc ma svetlo pro pozorovatele ze vzajemne se pohybujicich soustav stejnou rychlost.
Narazim na priklad s domlouvanim miru v jedoucim vlaku, kdy je uprostred zarovka, ta se rozsviti a statnici na obou koncich vlaku v okamziku kdy k nim dorazi svetlo podepisi smlouvy. Pro pozorovatele na nastupisti se to pri dostatecne rychlosti jevi tak, ze ten v predu to podepise drive. Je tam psano, ze na svetlo se neuplatnuje princip skladani rychlosti. Ale ja se ptam proc?

Svetlo je vlneni, ktere v gravitacnim poli meni smer, prechodem mezi prostredimi s nestejnymi indexy lomu se lame... takze ma "fyzickou podstatu". Proc pro rychlost svetla neplati to, ze kdyz zdroj svetla umistime na rychle se pohybujici vozik, tak je pro pozorovatele v klidu jeho rychlost dana rychlosti svetla+rychlost voziku?

Urcite kazdeho z fyziku toto napadlo a kazdy si to nejak vyresil, tak me to sdelte. Ja kvuli tomu "Vesmir" dal nectu, stale o tom premyslim... Diky za snahu s "nevzdelancem :-)"


Příspěvek od: Lubos Motl
Čas: 03:17 06.09.2001
E-mail: motl@physics.rutgers.edu
Web: http://www.matfyz.cz/lumo/
Dobreho dne! Zkusim zacit. Dik, pane Lomozi.

1. Ano, nekdy je ruznych moznosti, jak svinout rozmery apod., tolik, ze to skoro vypada, ze jsme si nepomohli. Ale vsechny tyto volby Calabi-Yauovych variet jsou jednotlive, diskretni, zatimco standardni model umoznuje menit parametry spojite. Tak tomu neni v pripade strun. Jakmile si ze spocetneho vyberu vybereme (spravne) vakuum - zpusob svinuti - vsechna spojita cisla lze presne spocitat. A navic jsou napohled ruzna vakua teorie strun propojena a dynamicky si vesmir vybere to jedine z nich - jednoduse receno to s nejnizsi energii. urcit, ktere to je, je ale nad dnesni sily.

2. V teorii strun je vsechno vyrobeno ze strun, svym zpusobem i Calabi-Yauova geometrie, z druhe strany struny potrebuji nekde zit, napriklad na Calabi-Yauove variete. Cioi v te teorii potrebuje jeden druheho i naopak, vsechno je propleteno. Takova je i filosofie dynamicke teorie. Teorie relativity ukazala, ze prostor a cas nejsou pevnymi objekty, ktere sice mohou ovlivnovat ostatni, ale nemohou byt jimi ovlivneny: podle obecne teorie relativity se prostor muze zakrivovat. Teorie strun jde v tomto smyslu do konce: neobsahuje vubec zadne objekty, ktere jen ovlivnuji ostatni, a samy nemohou byt ovlivneny. Vsechno je dynamicke.

3. Smesujete velikost objektu a velikost vlnoveho baliku. To jsou dve uplne odlisne veci. Struna je typicky velka asi tak jednu Planckovu delku (podle klasickych scenaru), ale to neznamena, ze jeji vlnovy balik musi nebo muze byt takhle maly! Je to jako s atomem vodiku. Ten ma desetinu nanometru: tohle je typicka velikost atomu. Ale jadro je jeste asi tak stotisickrat mensi a elektron nejmene stomilionkrat mensi. Elektron ma sice vlnovou funkci rozmazanou po cele velikosti atomu, ale on sam je mnohem mensi, miliardtinu miliardtiny metru nebo mene. Kdyz zkoumame jakymkoliv rozumnym zpusobem jeho strukturu, zjistime, ze je fakticky bodovy. To, ze vlnovy balik je rozmazany, je neco uplne jineho. Vlnovy balik je jen o pravdepodobnostech, ze najdeme elektron (ci strunu) tam nebo onde. Ale co tam najdeme, je vzdycky temer perfektne bodove v pripade elektronu (nebo jde-li o strunu, ma to velikost asi tak Planckovu delku, coz je take v podstate nic). Takze neplest!

4. Tenhle vyrok mne pripada perfektni. Cerna dira Planckovy velikosti je zaroven nejmensi cerna dira, na kterou se muzeme divat jako na cernou diru, zaroven nejvetsi mozny vazany stav elementarnich castic, u ktereho plati intuice, ze na zkoumani kratsich casu a vzdalenosti jsou treba vetsi frekvence (a energie): jeste vetsi energie nez Planckova uz neumoznuje studovat kratsi vzdalenosti, protoze zacneme vyrabet vetsi a vetsi cernou diru. Cerna dira Planckovy delky vazi asi Planckovu hmotnost a behem Planckova casu se vypari na Hawkingovo zareni, ktere ma samozrejme priblizne jednu Planckovu energii. ;-)


Příspěvek od: F. Lomoz
Čas: 20:34 05.09.2001
E-mail: flomoz@volny.cz
Vazeni panove, nyní procitam znovu Elegantni vesmir postupem nahodneho vyberu a dosel jsem k pozoruhodnemu poznatku. Kazda i mala část upouta a nuti vyhledavat souvislosti. Skoda, ze nam laikum není pristupny matamaticky aparat teotie.
Mam nekolik otazek:
1) Není snaha strunova teorie s hledanim vhodnych C-Y variet v nichz ziji struny (superstruny) podobna snaze standrdniho modelu dopatrat se podstaty vseho za cenu 19 volnych parametru?
2) Ptat se zda struny mohou vysvetlit sami sebe zrejme nemá smysl, jak je naznaceno v doslovu J. Langrem „…ptat se , z ceho struny jsou, nemá dobry smysl“. Presto musim polozit otezku. Je struna fyzikalni objekt, který potrebuje k realizaci C-Y varietu nebo naopak?
3) Energie fotonu je umerna frekvenci vlneni. Vyssi frekvence vyssi energie. Struna s delkou srovnatelnou s Planckovou delkou musí mit obrovskou frekvenci, aby se alespon jedna vlnova delka vesla do delky struny. Jaka finta strunove teorie zduvodnuje male energie (hmotnosti) castic, které jsou vibracnimi stavy strun?
4) V souvislosti se strunami a ve spojitosti se standardní teotii je prijatelna predastava, ze castice s Planckovou hmotnosti je nejmensi cernou dirou a zaroven nejtezsi „elementatni castici“?
Diky i za nevycerpavajici odpovedi.


Příspěvek od: Jirka P.
Čas: 23:06 17.08.2001
E-mail: jirka.p@seznam.cz
Dekuji Vam pane Motl, myslim ze vam rozumim. Vse
nasvedcuje tomu, ze to je opravdu pouze spatne
extrapolovana myslenka z pozorovani hologramu a
aplikovana jako pseudovedecka teorie. Lze nalezt
nejake dalsi usvedcujici info, aby to nebylo jen tvrzeni
proti tvrzeni? (tech. pozn.: entery mi tu dela browser
sam...:(


Příspěvek od: Lubos Motl
Čas: 15:16 17.08.2001
E-mail: lumo@matfyz.cz
Pane Jirko, je to dost osemetna vec. Ten clanek neobsahuje zadnou fyzikalni teorii. Je to laicke a zmatene povidani, kazda treti veta je nesmysl. Pokusy Alaina Aspecta v zadnem pripade neprotireci teorii relativity a neumoznuji sireni informace nadsvetelnou rychlosti, jak clanek zcela chybne tvrdi. Tohle je velmi oblibena, ale zcela nepravdiva interpretace, kterou laici radi kvantove mechanice (projevene v Aspectove experimentu) prisuzuji. Take neni pravda, ze experiment souvisi s holografii. David Bohm je vubec blazen a dal se uz pred desitkami let na kouzelnictvi (pote, co ho podvodnik Uri Geller presvedcil, ze umi delat telepatii a ohybat klice - Feynmana tento podvodnik rozhodne nezblbl). Od te doby Bohm dela vyslovene trapne veci (jako ze uz dost trapne delal i predtim - ty absurdni interpretace QM -, prestoze zacinal jako nadejny fyzik). Clanek pak rika neco napul spravneho o hologramech, ale zobecnuje je fyzikalne irelevantnim zpusobem. Karl Pribram je vyslovene blazen, ktery zije v komunite lidi, kteri zfetovavaji pomoci LSD sve obeti i sebe samotne a delaji ze sveho totalniho myslenkoveho zdecimovani vedu; viz tez Stanislav Grof a dalsi jmena v clanku. Strucne receno, ten clanek se snazi obsahnout jiste veci z fyziky, ale sam s fyzikou vubec nesouvisi, jak by se mnou jiste naprosta vetsina fyziku souhlasila. Neda se hodnotit "hypoteza" postavena na spekulativnim clanku, kde je kazda 3.veta nesmyslna.


Příspěvek od: Jirka P.
Čas: 19:21 16.08.2001
E-mail: jirka.p@seznam.cz
Dobra, prozatim Vam dekuji, zkusim se podivat na nejake dalsi informace o kterych jste se zminil a pripadne se na neco zeptam. Samozrejme by mne zajimaly nejake aspekty, ktere by se daly z faktu o holografickem principu vyvodit. Souhlasim s tim, ze v dnesnim modelu vesmiru musi mit struktura hmoty i informace na ni vazane nejaky limit. Nicmene rad bych tu precetl nejake jednoznacne prohlaseni, ze zminena teorie nemela uspech kvuli JASNE chybe v logice, chybe v praci s konstantami, ci nejake jine chybe.
Diky...:o)


Příspěvek od: Lubos Motl
Čas: 13:44 16.08.2001
E-mail: motl@physics.rutgers.edu
Web: http://come.to/lumo/
Zdravim! Ten clanek mne pripada lehce popleteny, micha holografii s pokusy Alaina Aspecta apod. Ale trvalo by moc dlouho, nez by se vsechny deformace vysvetlily. Holograficky princip je jinak opravdu dulezity v moderni teoreticke fyzice. Jde o to, ze oblast prostoru muze obsahovat jen tolik informace, kolik ji lze zakodovat na povrch tohoto prostoru s hustotou asi 1 bit na 10^{-70} metru ctverecniho. Viz napriklad http://www.kolej.mff.cuni.cz/~lmotm275/ruze/index2.html

O pokusech Aspecta jsem uz take par megabajtu napsal, viz napr. nekde na amber.zine.cz, ale nemam tu presne linky...
Pokusy Alaina Aspecta zcela potvrdily pravidla kvantove mechaniky, ktera sama o sobe vubec neimplikuje nic jako holografie. Snad bude mit nekdo vice casu na detailnejsi analyzu.


Příspěvek od: Jirka P.
Čas: 12:02 16.08.2001
E-mail: jirka.p@seznam.cz
Dobra, pripoustim, ze myslenka o takovem vesmiru je zatim vicemene teorii na filozofickem zaklade. Ale v clanku se pise take o nejakych vedeckych pokusech a take o vedeckych pozorovanich a nejakych zaveru z nich. Mam pocit, ze by se o tom nikdo moc nezminoval pokud by to bylo zalozeno pouze na uvaze filosofu... Myslim, ze se neco z dotycneho clanku nemusi vylucovat se soucasnou vedou, ackoliv to zatim neni meritelne. Muze se tu nekdo vyjadrit k 2-3. odstavci clanku? Ad Occamuva britva: Jakou metodou chcete neco pocitat ci merit, pokud by vlastnosti takoveho vesmiru lezely v nadmnozine vlastnosti soucasneho poznani vesmiru?


Příspěvek od: Vojta
Čas: 00:10 16.08.2001
E-mail: egg@matfyz.cz
Zdravim, Jirko!

Jeste si budu muset nekde najit, o co presne slo v tom zminovanem experimentu. Kazdopadne vim, ze jeste nikdo nepredvedl experiment, ktery by odporoval Einsteinove OTR. Proto mi diskuse, ktera se od toho odviji, pripada v podstate zbytecna.

Jak je teorie o holograficke podstate vesmiru znama verejnosti, to nevim, ja uz jsem o ni drive slysel. Muj nazor je takovy, ze diskuse o tom, jestli vesmir je "objektivni realita" nebo "holograficky prelud" patri do filosofie a ne do fyziky. Pro skutecneho fyzika je diky tzv. "Occamove britve" relevantni jenom to, co se da merit. Da se z diskutovani kolem holograficke podstaty vesmiru neco spocitat? Neco, co by se pak dalo zmerit? Obavam se, ze odpoved je ne.


Příspěvek od: Jirka P.
Čas: 22:50 15.08.2001
E-mail: jirka.p@seznam.cz
Prosim, rad bych se zeptal, jak dalece je znama mezi verejnosti teorie popsana na strance http://mujweb.cz/www/habib/vesmir.htm Pripadne jake k ni mate vlastni odborne stanovisko? Lze ji brat jako soucast jinych teorii, nebo naopak jine teorie soucasti teto? ...sam nejsem prilis uveden do poslednich vedeckych poznatku, pouze se o to zajimam. Omluvte proto mou pripadnou neznalost, nebo nevhodny vstup do diskuse.


Příspěvek od: DUREL Martin
Čas: 22:40 02.08.2001
E-mail: durel@atlas.cz
Vážený pane Motle, děkuji za upozornění co se jak píše. Netrvalo mě ani pět minut a našel jsem i Vaše překlepy. Např. : ( co na reknou ) ale to snad ne, že by snad takovému géniovi vypadlo slovíčko to? Ale přesto děkuji za pěkná přirovnání, které jste použil.
Martin D.



Příspěvek od: Lubos Motl
Čas: 05:47 29.04.2001
E-mail: motl@physics.rutgers.edu
Web: http://lumo.come.to/

Zdravim, pane Mikulko!

Gratuluji Vam k uspesnemu docteni knizky. Vase otazky jdou do cerneho. Zajimalo by me, co na reknou druzi -


Příspěvek od: Luboš Motl
Čas: 10:40 22.07.2001
E-mail: motl@physics.rutgers.edu
Pane Durele, nepodléhejte panice, nejste jediný, kdo z vědy "vypadl". Někteří neměli ani to štěstí, aby si udělali rodinu - myslím, že Vaše rodina si nezaslouží slůvka jako "bohužel". Procento bývalých a vyřazených vědců, kteří jsou profesionálně neúspěšní a nejenže se přestanou věnovat fyzice, jak píšete, ale i na vědu zanevřou a píšou pak svoje "moudré" postoje, není úplně zanedbatelné - a nejméně jeden další takový na toto fórum pod mnoha pseudonymy chodí. Je mně líto, že dnes máte s fyzikou společné jen to, že se setkáváte s lidmi, kteří Vám sdělují svá náboženská zjevení o tom, že vědci dělají nesmysly. Ano, je pravda, že dnešní poznání je mnohem dál, než se lze dočíst v populárních knihách. Ano, je asi pravda, že nejméně jedna věta i z této knihy bude opravena budoucím výzkumem. Není pravda, že teorii superstrun věří málokdo. Teorii superstrun bere vážně většina lidí, kteří ve fundamentální fyzice něco znamenají, a samozřejmě také většina laiků, kteří jsou s fakty obeznámeny. Různí sebevědomí lidoví koumáci jako Vy, kteří si myslí, že vědí všechno lépe než fyzici, tvoří tak 3 procenta lidí, kteří na Elegantní vesmír reagují - viz amazon.com. A pane Durele, slovo "dnešní" se píše bez "t".

Luboš


Příspěvek od: Martin DUREL
Čas: 18:45 21.07.2001
E-mail: Durel@atlas.cz
Motlovi
Asi týden jsem četl veškeré příspěvky a mám pocit, že dneštní pohled na vesmír je na samém začátku. Když jsem dostudoval a složil státní zkoušku z matematiky a fyziky tak jsem toužil se dostat do výzkumu a pokračovat v tom v čem jsem začal. Bohužel pak přišla rodina a dítě a moje další studium nebylo možné. Tak jsem začal dělat do počítačů. V té době to bylo velmi populární. Později jsem začal programovat a moje myšlení bylo zcela převedeno na algoritmy. Přestal jsem se úplně věnovat fyzice, jen matematiku jsem úspěšně využíval. Tohle až doposud není důležité. Nedávno jsem poznal člověka, který mě opět donutil se vrátit k fyzice. Tenhle člověk dokáže "předvídat budoucnost a vracet se zpět". Sám jsem tomu nevěřil až jsem se dostatečně přesvědčil o jeho schopnostech. Pokud toto vezveme za fakta, pak časová linie tak jak ji popisujete koliduje s tímto faktem! Po přečtení Elegantního vesmíru mám pocit, že je zde účelně zavedená teorie, která má zabránit skutečnému hledání správné teorie. Při představě, že tisíce schopných fyziků mrhá svou energií na hledání něčeho, čemu dnes málokdo věří, je děsivá. Možná je to až příliš propracované a má to odklonit dosavadní výzkum. Sám jsem přesvědčen, že ne všechno co bylo popsáno ve zmíněné knize bude pravda. Jen na závěr, dneštní vědecké poznání světa a technologie jsou mnohem dál, než se píše v knihách.

Martin D.


Příspěvek od: Vlasta(on)
Čas: 22:25 04.06.2001
E-mail: vpet@volny.cz
Motlovi,
Teď jsem nahlédl, odpověď jenom přelít. Chci se vyjádřit k té druhé části. Ale přesně tak jsem to přeci myslel!!! Obecně je víc věcí které nevíme než víme a zřejmě ještě víc těch o kterých ani nevíme že o nich nic nevíme :-) . Já jsem chtěl říct že tento způsob je sice poučnější a lépe „uvádí do problému“ ale zároveň „relativizuje“ otázky. A "relativita" je pro spoustu lidí ohromný problém :-) nebo :-( ?
Zatím díky.


Příspěvek od: Lubos Motl
Čas: 17:27 04.06.2001
E-mail: lumo@matfyz.cz
Web: http://www.matfyz.cz/lumo/
AdS je zkratka za Anti de Sitterova metrika, mimochodem jsem ji v Poincareho souradnicich napsal nize. Existuji tri euklidovske metriky s maximalni symetrii (kde kazdy bod je rovnopravny a kazdy smer od neho je rovnopravny): plochy prostor, sfera (povrch koule) a Lobacevskeho geometrie.

Analogicke tvrzeni pro casoprostory (s jednim casovym smerem) je, ze maximalne symetricke prostory jsou plochy casoprostor, de Sitteruv prostor (prostor s kladnou kosmologickou konstantou, v nemz asi - priblizne - zijeme) a Anti De Sitteruv prostor (s negativni kosmologickou konstantou, v nemz nezijeme). Oba dS a AdS si lze predstavit jako jiste hyperboloidy vnorene do casoprostoru s jednou dodatecnou dimenzi: napriklad dS4 - ctyrrozmerny de Sitter - lze vnorit do petirozmerneho prostoru se signarutou 4+1, zatimco AdS4 lze vnorit do 3+2-rozmerneho casoprostoru. Jen napis spravnou rovnici hyperboloidu, x1^2+x2^3+x3^2-x4^2-x5^2=-r^2... vysledna nadplocha je AdS prostor.

AdS prostor hral a hraje vyznamnou roli v teorii strun poslednich let, protoze Maldacena ukazal, ze gravitacni teorie v AdS prostoru lze popsat pomoci konformni teorie pole (CFT) zijici na hranici.

dS je naproti tomu jaksi konecny, ma horizonty podobne jako cerna dira (urcite udalosti jsou od nas navzdy oddelene), mel by mit konecnou entropii apod.


Příspěvek od: Vojta
Čas: 11:18 04.06.2001
E-mail: egg@matfyz.cz
Web: http://www.matfyz.cz/egg
Omlouvam se za mezery ve vzdelani, ale co je "AdS metrika"? To znamena, v 5D prostoru metrika na 4-brane?


Příspěvek od: Lubos Motl
Čas: 17:54 03.06.2001
E-mail: lumo@matfyz.cz
Web: http://www.matfyz.cz/lumo/
Jo jinak. Ja nemam moc pocit, ze kniha rika moc "nevime". Rika "nevime" tam,. kde nevime - Brian je nakonec seriozni vedec, ktery nemuze jen tak kecat -, ale jinak podle meho v podstate vysvetluje uplne vsechno dulezite, co o fundamentalni fyzice vime. ;-) Jestli nejaka knizka rika, ze vime mnohem vice apod., tak nemluvi pravdu. Mam dojem, ze jine knizky jsou mnohem vice fuzzy - plno spekulaci o multivesmirech, kosmickych mutacich, Bohu, stvoreni, nabozenstvi, souvislosti vedy s emocemi atd. atd. Tohle jsou casto veci, o nichz nejen vime, ze o nich moc nevime, ale vetsinou dokonce vime, ze to, co autori takovych knih "vedi", jsou blbosti. :-)


Příspěvek od: Lubos Motl
Čas: 17:48 03.06.2001
E-mail: lumo@matfyz.cz
Web: http://www.kolej.mff.cuni.cz/~lmotm275/forum/
Take jsem ocekaval o neco vetsi explozi :-), ale castecne je duvodem to, ze asi polovinu odcerpal konkurencni chat na http://www.kolej.mff.cuni.cz/~lmotm275/forum/ ...

Ja jsem z deni kolem Randall-Sundrum trochu roztrpcen. Existuje radove tisic clanku o predmetu, a pritom nejsou vyjasneny ani zakladni veci. Zvlaste kdyz se do veci pusti relativiste s jejich nastroji, zmateni muze byt dokonale.

Randallova a Sundrum napsali nekolik rozbuskovitych clanku souvisejicich s branovymi svety (brane worlds). Jde o to, ze ukazali, ze dodatecne dimenze prostoru vubec nemusi byt male a svinute, jak jsme si mysleli, ale mohou byt fakticky i nekonecne (my ovsem musime zit na brane, ktera nesmi mit krome znamych 3+1 dimenzi zadne dalsi velke dimenze). Napsali spolu asi 3 clanky, RS0, RS1 a RS2. V RS1 uvazuji, ze prostor muze mit velkou patou dimenzi, ackoliv casoprostor nemusi byt kartezskym soucinem. V tomto 5D prostoru je AdS metrika

ds^2 = L^2.dy^2 + exp(-2y) [-dt^2+dx^2+dy^2+dz^2]

Vsimnete si, ze je 4D metrika v hranate zavorce preskalovana "warp" koeficientem exp(-2y): v ruznych mistech "y" ve smeru pate dimenze vnimame ruzne vzdalenosti. L je polomer krivosti AdS. V RS1 uvazovali, ze "y" je definovano na usecce, ktera konci na jedne strane branou "y=0" s negativnim napetim (tam je standardni model, tam tedy ziji obvykle castice, elektron apod.) a na druhe strane branou s pozitivnim napetim (tam je soustredena vetsina vlnove funkce gravitonu), dusledkem toho je gravitace zeslabena, protoze o nas svet petirozmerny graviton jen "zavadi" exponencialne klesajicim ocaskem.

V RS2 ukazali, ze pata dimenze muze byt fakticky i nekonecna, pricemz v scenari je jen jedna brana, na niz zijeme. U svinutych dimenzi vetsinou rozvadime pole do modu typu exp(imx) (nebo sferickych funkci), ktere tvori diskretni spektrum. Castice na nekonecne care ma ale jak znamo spojite spektrum. Pridame-li vsak potencial, najdeme diskretni spektrum. Prave tento potencial pro graviton poskytuje zakriveni 5D prostoru. Dusledkem toho nakonec odvodime na brane gravitacni silu 1/r^2 jako v nasem prostoru, nikoliv 1/r^3 jako v prostoru o dimenzi vyse: graviton je "prilepen" k branam.

No a pak existuje asi tisic clanku, jak narusit SUSY, jak se pokusit vysvetlit nulovou kosmologickou konstantu, jak .... a potom take pokusy realizovat scenar RS jako model v teorii strun - Verlinde nejdrive prisel s geometrii blizko horizontu nejakych D-bran v teorii strun, Polchinski et al. nedavno navrhli, jak muzeme zaclenit RS do F-teorie. Ale jinak mam z deni kolem RS dojem chaosu, ackoliv puvodni myslenka je jiste zajimava. Z RS se take zatim nevyklubala dospela alternativa pro reseni problemu hierarchie. Michale, oprav me, jestli v necem kecam.


Příspěvek od: Vlasta(on)
Čas: 15:31 03.06.2001
E-mail: vpet@volny.cz
Nejdříve bych chtěl říct, že jsem se mýlil ve svém mailu Vojtovi, že po vyjití EV se rozhýbe tento chat. EV v mnohem větší míře než jiné, obdobně zaměřené knihy, zdůrazňuje co nevíme, než co „víme“. Na co se potom má čtenář ptát, když již dostal odpověď – nevíme. Prosím, to není kritika, to je konstatování prosté skutečnosti. Myslím že OTR byla v EV vysvětlena opravdu tak dobře, že i pan Zbytovský pochopil, že „tyč může odplavat do kanálu“. Co mě však na EV opravdu zaujalo, byla (tedy podle mého omylného názoru) lehkost s jakou dokázal autor (možná i překladatel – neumím posoudit) hned v úvodu navodit zájem o jednotlivé částice a tedy kvantovou mechaniku - teorii strun a OTR pojmout jako jakousi „doplňkovou“ teorii, jakousi „newtonovskou fyziku“. Omlouvám se za ne zcela nejpregnantnější vyjádření. Předpokládám že IQ účastníků chatu umožní pochopit co chtěl autor říci.
Nyní mě odpusťte ten úvod a dovolte mě přednést jednu spíše žádost. (Obávám se, že se nejspíše bude týkat pouze Motla a Fabingera.) Dalo by se nějak shrnout "dění" okolo Randall – Sundrum scénáře a "warpovaných rozměrů"? Může tam být i nějaká ta „rovnička“ ale prosím s popisem. Ne vždy hned vím co označuje co a „luštění“ je těžká dřina na jejímž konci si jeden připadá jako blbec. Ne že by to muselo být tak daleko od pravdy ale je dost lidí kteří mě to jsou ochotni kdykoliv sdělit, že sám sebe koukám šetřit.

Díky.


Příspěvek od: Jan Kocanda
Čas: 12:58 23.05.2001
E-mail: koc12@post.cz
Kniha EV je proste skvela. Dik pane Motl.
Jan Kocanda


Příspěvek od: František lomoz
Čas: 06:45 20.05.2001
E-mail: f.lomoz@worldonline.cz
Zdravím diskutující.
Shledávám, že EV je zdrojem inspirace. Dočetl jsem se o specifickém vibračním módu strun odpovídající jediné částici, gravitonu. Z předchozích příspěvků je snad patrný náznak mého vlastní přístup k pohledu na mikrosvět, který je asi superstrunný. V tomto pohledu používám pro „cestování“ ve světě strun číselný parametr v mezích od nuly do jednotky, kterým ovládám moment setrvačnosti struny „d*m*R^2“. Při podmínce vynulování vibračních modů a vyjádření leptonů a kvarků jako rotačních modů strun nachází se parametr d v okolí 1/12 pro leptony, v okolí ½ pro dolní kvarky (d) a v okolí 1 pro horní kvarky (u). V prvém případě toto připomíná struny s otevřeným koncem (ale není to nutné), ve druhém případě to jsou spíše plošné tvary (např. kruh) a ve třetím případě pak struny uzavřené (kružnice).
Překvapilo mne, že existuje určité konkrétní řešení při kterém je parametr d libovolný. V tomto případě jsou superponované vibrační mody ve svém jednom sektoru nulové a přitom rotační mód dává jediné řešení. Přiléhavé je spojení s oblastí Higgsových bosonů.
Při zmínce o gravitonech jsem hledal podmínku jejich existence. Překvapilo mne jak je jednoduchá. Zakázal jsem rotační stavy strun. Vyplývající vibrační stav dává základní vibrační hladinu, kterou lze asi přiřadit gravitonu. Vykazuje dvě shodné „amplitudy“ a dvě komplexně sdružené „dvojice fází“.
Samozřejmě, že mezi těmito popsanými stavy (do jisté míry krajními) existuje nepřeberné množství exotických vibračně-rotačních stavů. Specifické stavy pak mohou odpovídat kvantům mnohých polí apod.
Teorie strun jistě dokáže rozlišit druhy částic. Zajímalo by mne, co odlišuje aspoň v hrubé podobě např. fermiony od bosonů se spinem 1?
Dobré celé další dny přeje F.L.


Příspěvek od: mrusek
Čas: 01:46 20.05.2001
E-mail: mrusek@volny.cz
Web: http://casodej.hyperlink.cz
Jasné pohledy ahoj :-)

Myslím, že počet dimenzí existujícího prostoru je značně závislý na právě probíhajícím kontaktu s těmito dimenzemi. Např. obyčejný 3 rozměrný prostor mizí, když se v něm přestaneme pohybovat a přestaneme vnímat prostorové signály.
Zdá se, že podobně se mohou objevovat a mizet i ostatní dimenze. Myslím, že celá teorie je právě o přechodech mezi jednotlivými stavy. Myslím též, že je dobré znovu a znovu upozorňovat na fenomén rekurze, kdy právě vznikající síla začiná zpětně působit sama na sebe. V tomto smyslu lze některé síly vnímat spíše jako negativní, jiné zase jako pozitivní. Je zajímavé sledovat, co je kritériem. Narážím v tomto okamžiku na značnou mnohost přístupů k tomuto tématu. Nemáte někdo radu? (Do pošty prosím, nevím, kdy tu budu.)

Díky Michal Rusek, Samo-tišky

P.S.: Abych upřesnil. Myslím, že to hodně souvisí s označováním jednotlivých jevů. Možná budeme v blízké budoucnosti svědky nového druhu značení sil, které velmi často působí, a právě nové označení pomůže celému systému najít tak dlouho hledané dodatečné množství energie. Mám představu, že verbální označení existujících sil je mnohem více podstatné, než si možná myslíme. Na druhé straně je dost dobře možné, že stačí již pouhé vědomí existence nového označení, jež není zatížené minulými komplikacemi, k tomu, aby i starý systém *ihned* převzal všechny nové vlastnosti.


Příspěvek od: Michal Rusek
Čas: 01:28 20.05.2001
E-mail: mrusek@volny.cz
Web: http://casodej.hyperlink.cz
Pane Krotky :-)

vidim, ze Vas take kniha zaujala velmi,
podobne jao mne.

:-)


Příspěvek od: Lubos Motl
Čas: 17:42 10.05.2001
E-mail: motl@physics.rutgers.edu
Web: http://lumo.come.to/
Vazeny pane Krotky, gratuluji k precteni, dekuji za pochvalu a zajimave otazky, byt formulovane ponekud lidovym zpusobem. Jsem rad, ze knihu ctou lide vsech moznych i nemoznych "socialnich" skupin.

1. Ne, mozna Vas zklamu, ale nehulim (mam k takovym vecem odpor) a nemyslim si, ze je to prave zdrava lidska aktivita, ackoliv znam lidi, kteri rikaji i cini opak - a obhajuji treba nazor, ze takova marihuana mozku vubec neskodi apod.

2. Myslim si, ze Einstein byl v celkovem pohledu originalnejsi myslitel i nez Witten (mozna jen zil v lepsi dobe? Nevim...), ale po strance schopnosti ovladnout slozite technicke otazky v matematice i fyzice, po strance vykonnosti i po strance vstrebat nejnovejsi pokroky v oboru, ktere jsou objevem jinych, je Witten o mnoho trid pred Einsteinem. Take jsem presvedcen (a ne sam), ze Newton byl o neco vetsim geniem nez Einstein.

3. Sam urcite nejsem clovekem, ktereho by meli radit do prvni desitky ani stovky. ;-) Fyzika neni tenis, a tak asi neni vhodne rozebirat, jestli je nekdo podle nekoho na 150.miste zebricku v originalite a genialite nebo na 400.miste v celkovych prispevcich k oboru. :-)

Preji hezky den
Lubos


Příspěvek od: Jan Krotky
Čas: 13:36 10.05.2001
E-mail: drkrotky@post.cz
Pane Motle,
precetl jsem si Vas stavnaty a perfekcionisticky preklad knihy El. vesmir a mel bych k Vam nekolik dotazu:

1.Hulite ?
2.Myslite si, ze je Witten v poradi geniality pred Einsteinem nebo az za nim ?
3.Kam se v tomto zebrizku radite Vy ?

Zdravi
Jan Krotky


Příspěvek od: Michal Fabinger
Čas: 01:47 01.05.2001
E-mail: fabinger@stanford.edu
Pane Lomozi, nevim, co presne myslite temi degenerovanymi rozmery.Nicmene pokud se nemylim, tak v principu neni vylouceno, ze tri prostorove rozmery jsou ty nase bezne nekonecne rozmery, tri jsou svinute a tri jsou "warpovane" stejne jako v Randall-Sundrumove scenari, o kterem zde uz byla rec. Tim mam na mysli, ze jsou sice nekonecne, ale ze se efektivne chovaji jako konecne. Muzete o nich tedy rict, ze jsou v urcitem slova smyslu degenerovane.

Zdravi Michal


Příspěvek od: Lubos Motl
Čas: 23:13 30.04.2001
E-mail: motl@physics.rutgers.edu
Web: http://lumo.come.to/
Preji hezky den!

Otazku druhou necham asi uplne na nekom jinem, ale zkusim zacit s otazkou prvni od pana Lomoze (mimochodem diky za pochvalu apod.). Snad me nekdo opravi a doplni, pokud neco reknu spatne nebo nesrozumitelne.

Elektricky naboj muze mit podle teorie strun nekolik ruznych puvodu. Tak treba v teorii strun typu I existuji tzv. otevrene struny, ktere maji dva konce; konec takove struny se chova jako kvark nebo antikvark, ktery nese urcity naboj.

V ostatnich teoriich strun je ale naboj plynule rozprostren po cele strune (uzavrene, ve tvaru smycky bez koncu). Velikost naboje je spjata presne se zpusobem, jak struna vibruje. Presneji receno, struna muze vibrovat v ruznych smerech a na ruznych frekvencich (frekvenci zde minim pocet uzlu podel struny). Energie kazdeho takoveho druhu vibrace je podle kvantove mechaniky kvantovana, tedy nasobkem nejake zakladni nominalni hodnoty. A napriklad mnozstvi kvant energie, ktera prameni z vibrace struny ve skrytem rozmeru XY, se projevi jako elektricky naboj. Pripadne se jako elektricky naboj muze projevit celkove navijeci cislo (kolikrat je struna navinuta kolem nejakeho rozmeru nebo kolem "diry" ve skrytych dimenzich, pripadne kolikrat je brana o vyssi dimenzi nabalena) nebo celkova hybnost ve smeru nejake dimenze, ktera je take kvantovana.

V osmdesatych letech lide verili, ze jedina teorie strun, ktera muze popsat nas svet, je heteroticka E8 x E8 teorie, v niz je elektricky naboj urcen tim, kolikrat je struna navinuta kolem jedne z 26-10=16 dodatecnych dimenzi, ktere existuji jen pro vlevojdouci mody strun (viz jedna z poznamek na konci knihy, obsahujici cislo 26). Toto navijeci cislo se zaroven rovna hybnosti.

Presne detaily zaviseji na konkretnim modelu, ktery studujeme, ale vzdycky plati, ze ma elektricky naboj interpretaci v nejake vlastnosti vibrace dane struny. Cim vice strunu rozvibrujeme, tim vetsi vetsinou ponese i naboje.

S pranim hezkeho dne
Lubos


Příspěvek od: František Lomoz
Čas: 22:19 30.04.2001
E-mail: f.lomoz@worldonline.cz
Vážení diskutující,
přečetl jsem EV jedním dechem a přiznávám, že velmi rychle. Dobře psaný text a dobře přeložený k tomu nutí. Pochopení podstaty věci je nepřímo úměrné rychlosti četby, ale celkový dojem je skvělý. Každý se bude k této knize vracet a konečně po kouskách a na přeskáčku ji přečte pozorně.
K mnoha-rozměrovým prostorům ve kterých vládnou struny mám dvě otázky.
1) Z jakého principu vyplývá náboj částice v rámci strunové teorie? Jede o druh vibračního stavu nebo je to jinak.
2) Je možné chápat současný makrosvět se třemi rozměry (+čas) i tak, že sice existuje devítirozměrný prostor, ale na pozadí třech makrorozměrů jsou další tři makrorozměry (tedy jakási degenerace rozměrů) a další tři jsou pak svinuté rozměry?
Pozn.: Degenerované rozměry jsou od sebe nerozlišitelné, avšak na úrovni např. horizontů černých a bílých děr degenerace vymizí. Na úrovni běžných gravitačních objektů pak tuto degeneraci lze chápat jako superpozici potenciálních černých a bílých děr.

Zdraví F.L.


Příspěvek od: Lubos Motl
Čas: 05:47 29.04.2001
E-mail: motl@physics.rutgers.edu
Web: http://lumo.come.to/
Zdravim, pane Mikulko!

Gratuluji Vam k uspesnemu docteni knizky. Vase otazky jdou do cerneho. Zajimalo by me, co na reknou druzi - prinejmensim Michal Fabinger, pokud uz sem neprijde nikdo z ceskych univerzit apod. - ale muj pohled je asi nasledujici:

1. Hlasky astronomu ve mne skutecne vyvolavaji duveru, ze existuje nenulova (kladna) kosmologicka konstanta (jakasi hustota energie ve vakuu), pripadne ma zrychlujici se rozpinani puvod v necem, co se kosmologicke konstante sakramentsky podoba. Jeji hodnota je ale silene mala, ve srovnani s typickou hustotou energie v teorii castic (neco jako hustota atomoveho jadra). Dnes tedy v podstate celime "dvema" problemum kosmologicke konstanty: jeden je ten, proc neni velka (jak by plynulo z jakychkoliv prirozene vypadajicich vysledku), a druhy je v tom, proc je presto nenulova.

Zodpovezeni techto otazek a rozumnejsi spocteni kosmologicke konstanty patri podle meho nazoru mezi nejvetsi resty nejen teorie strun, ale casticove fyziky vubec. Je to tuhy problem, ale zaroven musim rict, ze clanky, ktere o kosmologicke konstante podle strun, kvintesenci (coz znamena neco jako pomalu se menici kosmologicka konstanta) apod. vychazeji, pusobi depresivnim dojmem (nejen na me) a IMHO neprinaseji zadne jasne poznatky, byt jsou takovych clanku desitky. Nepochopeni kosmologicke konstanty je jedna z velkych ostud teorie strun a teoreticke fyziky vubec. Jednou snad nekoho napadne neco opravdu chytreho... Z druhe strany presto, ze nase stavajici teorie (i struny) davaji uplne spatnou predstavu o kosmologicke konstante (predpovidaji, ze hustota hmoty ve vakuu by mela byt v podstate stejna jako hustota atomoveho jadra nebo neco podobneho), nezahazoval bych kvuli tomu teorii strun. :-) K.k. predstavuje vec, ktere nerozumime, a tak by si tato vec nemela moc vyskakovat a delat si narok ovlivnovat poznatky, kterym rozumime nesrovnatelne lepe.

2. Gravitacni experimenty, ktere meri chovani gravitace na submilimetrovych vzdalenostech, jsou v plnem proudu a celkem pravidelne slychame prednasky. Minuly tyden treba mluvil jeden dost sofistikovany experimentator o rezonancnim experimentu, pri nemz se otaceji dva disky s 26 derami vyriznutymi podle obvodu kazdeho z disku. Tyto diry zpusobuji periodicky se menici gravitacni silu, kterou jde merit (otresy, rezonance...). Experimentatori dosli uz nekam k 75 mikrometrum (vzdalenost mezi hmotami) - vezmete si, jak prtava je gravitace mezi 75mikrometrovymi kousky hmoty - a zadne odchylky od Newtonova zakona nenasli, coz znamena, ze prilis revolucni hypotezy o dimenzich velkych 1mm apod. bychom meli zapomenout; limit jde trochu nize. Experimentatori rikaji, ze budou schopni dojit asi tak k 10 mikrometrum. Nadeje, ze overit odchylky od Newtonova zakona a najit nove dimenze timto lacinym zpusobem, tedy klesa. Na urychlovaci LHC ale budou mit experimentatori jiny druh nastroju, jak pripadne extra dimenze najit.

3. Dik, autor by mel jiste radost (ted zrovna odepsal na prvni zpravy o prodeji apod.). Kdybych napsal knihu ja, asi bych si netroufl ji nazyvat "pokracovanim" EV :-). Z druhe strany za prava na dalsi knihu Briana Greenea zaplatili vydavatele Knopf a Vintage na Frankfurtskem kniznim veletrhu 2000 presne 2 miliony dolaru, coz byl nejvetsi obchod (druha v poradi byla Posh Spice s memoary, 1,5 milionu). Predbezny nazev je "The Fabric of the Cosmos: Space, Time and the Texture of Reality", nic vice nevim. IMHO existuje i dost velky zajem o knihu technicky o neco narocnejsi (neco mezi Elegantnim vesmirem a Polchinskeho ucebnici String Theory), ale pochopitelne takova vec by se mohla stat bestselleremu hure. :-)

Myslim, ze pokroky v teorii jsou a budou, ale revoluce, kterou jsme prozili pred par lety, byla prece jen vyjimecna doba a ze tolik se toho stane tak za 10 nasledujicich let. Takze bych to s tou frekvenci srozumitelnych reportu neprehanel. Lide od fochu samozrejme museji sledovat, co se deje, v podstate kazdy tyden, ale nebojte, pokud Vam neco utece, ne vice nez na rok. :-)

Preji hezky den
Lubos


Příspěvek od: Pavel Mikulka
Čas: 02:29 29.04.2001
E-mail: mikulka@nspuh.cz
Přečetl jsem knihu Elegantní vesmír a rád bych se zeptal na pár otázek.

1. Zda urychlení rozpínání vesmíru tak, jak to dokazuje od roku 1996 pozorování supernov, a jak se píše na str. 345 znamená, že kosmologická konstanta bude mít malou nenulovou hodnotu, nebo zda jde o nový, dosud nepopsaný jev v historii vesmíru. Poku jde o nový jev, co tato tajemná akcelerační síla udělá s teorií superstrun ?

2. Kdy proběhnou gravitační experimenty, které by mohly potvrdit existenci velkých svinutých rozměrů ?

3. Protože se mi velmi líbí celá kniha a hlavně její závěrečné kapitoly, které nastolují řadu otázek a upřímně odpovídají, že fyzici jsou na počátku velmi složité cesty, rád bych věděl, zda se dočkáme za pár let pokračování knihy, a to tam, kde kniha Elegantní vesmír končí. Možná, že by mohl pan Motl nebo jiný český fyzik uveřejňovat vždy po jisté době krátký článek v nějakém populárně vědeckém časopisu, kde by přijatelnou formou popsal další pokroky v superstrunové teorii.


Pavel Mikulka


Příspěvek od: Michal Fabinger
Čas: 23:28 06.04.2001
E-mail: fabinger@stanford.edu
Frantisku Lomozovi: Coze??? Vsechny superstruny maji vzdy excitovane nejake (vibracni, pokud chcete) mody, a to vcetne tech strun, ktere maji nenulovy spin (vnitrni moment hybnosti).


Příspěvek od: Vojta
Čas: 22:19 06.04.2001
E-mail: egg@atlas.cz
> Jestliže si superstunová teorie klade za cíl sjednotit vše, pak do diskuse patří také vše.

:-)) No vlastne mate pravdu. Ale nemusime se tu prece zabyvat vecmi, ktere jsou uz uspokojive vyreseny v jednodussich teoriich, ktere ze superstrun vyplyvaji v nejake limite. Natozpak vecmi, ktere vyplyvaji z definic fyzikalnich jednotek! Rozumite mi, ne? Jinak tem vibracnim a rotacnim modum ja nerozumim, to at komentuje nekdo kvalifikovanejsi.


Příspěvek od: František Lomoz
Čas: 20:44 06.04.2001
E-mail: f.lomoz@worldonline.cz
Nerad bych diskusi zaneřádil blbostmi a odbočujícími úvahami. Jestliže si superstunová teorie klade za cíl sjednotit vše, pak do diskuse patří také vše. To je asi taky blbost.
Vrátím se tedy k odsouhlasenému. Struny mají rotační mody. Přidal jsem k tomu domněnku, že trojsložkový rotační stav podmiňuje nulovost vibračních modů. K tomu abych stanovil pro jaké tři složky je vibrační mód nulový, postačuje znát poměr hmotností příslušných částic, které jsou výsledným produktem. Např. poměr hmotnosti mionu k hmotnosti elektronu nebo poměr s kvarku k d kvarku nebo poměr c kvarku k u kvarku nebo poměr mionového neutrina k elektronovému neutrinu.


Příspěvek od: Lubos Klouda
Čas: 14:20 04.04.2001
E-mail: glin84@post.cz
Zdravim,

Ac se to netyka teorie strun, coz jiste ponesete se znacnou nelibosti, rad bych upozornil, ze napriklad v bezmagnetickem studenem plasmatu je relativni permitivita
elektromagnetickych vln vzdy mensi nez 1. Fazova rychlost EM vln je tedy vetsi nez C. Je pravdou, ze vektor polarizace ma opacny smer nez vnejsi elektricke pole - je to dano pusobenim okolnich nabitych castic.

Lubos


Příspěvek od: Lubos Motl
Čas: 23:26 03.04.2001
E-mail: motl@physics.rutgers.edu
Web: http://lumo.come.to/
Mily pane Lomozi,

rad bych Vas potesil nejakym lepsim ohlasem na Vase navrhy na vypocet hmotnosti castic, nez kterymi Vas obstastnili Michal a Vojta, ale bohuzel me nic nenapadlo. ;-)

Vojta pravdu di o definici metru a sekundy (pred rokem 1980 byl i metr definovan pomoci vlnove delky jineho zareni, ale to je minulost a je z fyzikalniho hlediska mile, ze "c" je konecne konstanta, kterou zname presne, protoze dulezitost "c" jiste prevysuje dulezitost nejakeho konkretniho zareni konkretniho atomu). "Vas" vztah epsilon0.mi0.c^2=1 je hezka vec, ale souvisi spise s volbou jednotek. V SI uzivame jednotky, ktere naziraji na elektrinu a magnetismus jaksi rovnocenne (epsilon0,mi0), a proto muzeme spocitat z techto konstant rychlost, kterou se pohybuji elektromagneticke vlny, a vyjde nam "c". Mimochodem v SI je mi0 presne rovno 4.pi.10^-7 prislusnych jednotek, a jelikoz je rychlost svetla 299792458 m/s, take epsilon0 ma presnou hodnotu, podle definic a podle Vaseho vztahu.

V jinych soustavach - zvlaste v Gaussove CGS - se uzivaji jine jednotky pro elektricke a magneticke veliciny, konkretne tam neni zadny amper apod., ale vsechny jdou vyjadrit jako souciny mocnin centimetru, gramu a sekundy, a proto z nich nemuzeme zadnou rychlost svetla spocitat: je zasazena primo do elektromagnetickych rovnic jako parametr. V soustave SI Maxwellovy rovnice zadne "c" primo neobsahuji, obsahuji ho neprimo diky vztahum D=epsilon.E a B=mi.H, epsilon0.mi0.c^2=1.

Take Vam kluci jiste spravne pripomneli, ze zatimco "c" je rychlost pohybu elektromagnetickych vln ve vakuu, stejnou rychlosti se pohybuji take vlny gravitacni a jine nehmotne castice apod. a tuto rychlost nelze prekonat zadnym fyzikalnim vzruchem. Pro fyziku (vlastnosti casu a prostoru) hraje "c" zcela fundamentalni ulohu a jiste neni pravda, ze vliv konstanty "c" je omezen jen na svetlo a elektromagneticke jevy: je totiz zcela univerzalni, nehlede na to, ze se z historickych duvodu "c" nazyva "rychlost svetla" (protoze svetlo byla prvni vec, o niz lide vedi, ze se takhle rychle pohybuje). Nazev "Newtonuv gravitacni zakon" take neimplikuje, ze gravitaci se ridi jen telo Isaaca Newtona.

K Vasi otazce. Existuji prostredi, kde je mi vetsi, i prostredi, kde je mensi nez mi0, jsou to diamagnetika resp. paramagnetika (plus feromagnetika). Z druhe strany neexistuje prostredi, v nemz by byla permitivita mensi nez epsilon0. To by znamenalo, ze vektor polarizace ma opacny smer nez okolni elektricke pole, tedy ze zaporne nabite castice jsou "tahany" na opacnou stranu, nez ocekavame z Coulombova zakona. Tahle jemna asymetrie mezi elektrinou a magnetismem (mezi kterymi jinak panuje dost velka symetrie) souvisi s tim, ze existuji elektricke naboje, ale neexistuji magneticke naboje (monopoly, osamoceny sever od magnetu).

V jakemkoliv prostredi, kde je interpretace elektromagnetickeho vlneni jasna, je nakonec 1/epsilon.mi mensi nez c^2. Normalni svetlo se pohybuje ve vsech prostredich krome vakua pomaleji nez ve vakuu.

Zdravi
Lubos


Příspěvek od: Vojta
Čas: 22:56 03.04.2001
E-mail: egg@atlas.cz
V soustave SI je jedna sekunda definovana, jako trvani nevim ted kolika period zareni, ktere vyda urcity izotop Cesia pri prechodu mezi dvema presne definovanymi energetickymi hladinami. Jeden metr je definovan jako vzdalenost, kterou uleti svetlo za 1/299792458 sekundy. Takze rychlost svetla je presne (definitoricky) 299792458m/s. Neni tedy definovana zadnym podobnym vztahem, jako uvadite. Ten urcije spise vztah mezi permitivitou a permeabilitou.

Otazku existence prostredi s relativni permitivitou resp. permeabilitou mensi nez 1 ted nedokazu zodpovedet, aniz bych to nekde hledal. Kazdopadne se Vase prispevky, pane Lomozi, ani trochu netykaji teorie superstrun, coz nesu se znacnou nelibosti...


Příspěvek od: František Lomoz
Čas: 22:30 03.04.2001
E-mail: f.lomoz@worldonline.cz
Malý doplněk. Rychlost světla je definovaná elektromagnetickými vlastnostmi vakua rovnicí:

c = 1/odmocnina(epsilon*mý)

,kde epsilon (správněji epsilon s indexem nula) je permitivita vakua a mý (správněji mý s indexem nula) je permeabilita vakua. Existují prostředí v nichž je epsilon větší než epsilon-nula např. sklo, voda a mnoho dalších. V těchto prostředích je rychlost světla menší než vakuová rychlost. Otázka tedy zní: mohou existovat prostředí jejichž epsilon je menší než epsilon-nula? Tutéž úvahu lze spojit s permeabilitou.


Příspěvek od: Vojta
Čas: 12:09 03.04.2001
E-mail: egg@atlas.cz
Mala oprava: podle poslednich zprav vyjde EV uz 18. dubna.


Příspěvek od: Michal Fabinger
Čas: 01:17 03.04.2001
E-mail: fabinger@stanford.edu
Nejen, ze je to odvazne, ale hlavne je to pekna blbost. Nevim, co jineho se da na to rict.

Zdravi Michal


Příspěvek od: Vojta
Čas: 22:55 02.04.2001
E-mail: egg@atlas.cz
:-)))))
To mi pripada hodne odvazne...

Mimochodem, bylo uz tady receno, ze EV konecne vyjde 21. dubna? Ja osobne se moc tesim. A myslim, ze i tady se to vice rozbene.


Příspěvek od: František Lomoz
Čas: 22:04 02.04.2001
E-mail: f.lomoz@worldonline
Pozoruji, že se diskuse nehýbe, proto přidám námět z jiné strany Rubikovy kostky.
Přístup k řešení hmotnosti elementárních částic pomocí kvantově-rotačních stavů primárních částic zprostředkovaných přes zdrojové hmotnosti mě dovedl ke stanovení klidových hmotností kvarků. Kvark d by měl mít hmotnost 153x a kvark u 165x větší než hmotnost elektronu. Toto je očividně v nesouladu s přijímaným odhadem hmotnosti těchto kvarků řádově na 10 MeV což je zhruba 20x hmotnost elektronu.
Abych uvedl do souladu zmíněné hmotnosti kvarků s experimentálním odhadem a zároveň vypočítal hmotnost protonu a neutronu, musel jsem přijmout zajímavý předpoklad.
„V nádobě ve které jsou kvarky uvězněny v protonu či neutronu panuje záporná teplota“.
Přitom kvadrát efektivní hmotnosti kvarku má hodnotu:

M^2 = m^2 * (k*T/m*c^2 + 1)^2

Tato formule říká, že při určité záporné teplotě T je efektivní hmotnost M nulová, má-li klidová hmotnost částice velikost m. V rovnici k znamená Boltzmannovu konstantu a c je rychlost světla.
Moje diskusní otázka. Může existovat takové prostředí v nitru hadronů jehož teplota je záporná?
Sám si odpovídám takto: prostředí ve kterém předpokládám zápornou teplotu mohu chápat i tak, že zde připustím rychlost větší než je rychlost světla, avšak hraniční rychlost světla ve vakuu souvisí elektromagnetickou povahou, zatímco prostředí v hadronech má povahu gluonovou.
Potom vnější elektromagnetický svět je charakterizován termodynamikou kladných teplot a vnitřní uvězněný svět gluonový je charakterizován termodynamikou záporných teplot.


Příspěvek od: František Lomoz
Čas: 20:46 16.02.2001
E-mail: f.lomoz@worldonline.cz
Pánové asi máte pravdu. Nechal jsem se pouze ovlivnit tvrzením z teorie Fourierovy transformace ve které se odvozuje jistá nerovnost, která říká „ čím více je funkce soustředěna okolo svého středního argumentu, tím méně je soustředěna okolo svého středního argumentu její Fourierova transformace“. Jinými slovy: Čím kratší je impuls, tím roztaženější je jeho spektrální funkce (tím širší je jeho spektrum). Tomuto jevu se říká princip neurčitosti. Ve fyzikálním světě kvantové mechaniky jde o Heisenbergův princip neurčitosti. Tyto věty jsem nalezl v jedné knížečce o Fourierových transformacích. Bylo chybou je aplikovat na chybnou představu rozpadu sterilního neutrina.

Ještě krátká poznámka. V experimentech hledajících oscilace neutrin se používá měřitelný parametr, kterým je rozdíl kvadrátů klidových hmot těchto částic, se zápornou hodnotou. Takže, formálně vyšší neutrino má menší hmotu.


Příspěvek od: Vojta
Čas: 23:34 13.02.2001
E-mail: egg@atlas.cz
Lumo to zas jednou poslal driv nez ja :-)
OK. Jsem rad, ze jsme se na nepravdivosti toho prispevku shodli...


Příspěvek od: Vojta
Čas: 23:20 13.02.2001
E-mail: egg@atlas.cz
??????
Muze mi to nekdo prelozit? Fourierovu transformaci znam a myslim, ze tu nic neresi - nevyplyva z ni zadne fyzikalni tvrzeni. Rozpadem castice vznikaji nejake jine castice. Dejme tomu, ze rozpadem sterilniho neutrina vzniknou nejake fotony. Chcete rict, ze jich bude tolik, ze budou vyhovovat spektralnimu slozeni reliktniho zareni? To by asi muselo to neutrino mit o mnoho radu vetsi energii nez 0,000235 eV.... Nebo jsem to spatne pochopil?


Příspěvek od: Lubos Motl
Čas: 23:12 13.02.2001
E-mail: motl@physics.rutgers.edu
Web: http://come.to/lumo/
Dik, Michale, za doplneni. Ano, energie se z teploty hrube prepocitava jako E=kT, maximum ma zareni o teplote T nekde na energii fotonu #.kT, kde # je pomerne hnusna numericka konstanta radu jedna. ;-)

Pane Lomozi, nechci Vam do toho rypat, ale pravdive tvrzeni je jen to, ze neperiodicky, casove omezeny impuls ma Fourierovu transformaci obsahujici vsechny frekvence.

Neni pravda, ze ma Planckovo spektrum dolni hranici. Tepelne zareni o teplote T obsahuje VSECHNY mozne vlnove delky. Maximum je soustredeno kolem E=kT=hf=hc/lambda.

Rozpad castice nemuze byt neperiodickym impulsem. Castice v klidu ma naprosto jasno urcenou energii a diky zakonu zachovani energie se rozpada na castice o uplne stejne uhrnne energii, pokud jsou to treba dva fotony (jako o pionu), je vlnova delka techto dvou fotonu naprosto jednoznacne urcena.

I kdyz mate rozpad na vice castic - napriklad neutron na elektron, proton a antineutrino - spektrum energii neutrina napriklad je spojite v urcitem intervalu, ale ani vzdalene se nepodoba Planckove krivce, uz proto, ze Planckova krivka je nenulova pro vsechny frekvence, zatimco energie neutrina z rozpadajiciho se protonu nemuze byt tezsi nez rekneme 2 hmoty elektronu.

Sterilni neutrino nejspise neexistuje, ale i kdyby existovalo, neni duvod predpokladat, ze je nejlehcim z neutrin, ani ze je excitovanym stavem. "Vyssi kvantovy stav s nizsi energii" je protimluv. Take je protimluv, ze fermion s nejnizsi moznou energii ma urcitou pravdepodobnost rozpadu, to neni mozne.


Příspěvek od: František lomoz
Čas: 22:04 13.02.2001
E-mail: f.lomoz@worldonline
Planckova křivka tepelného záření představuje spojité spektrum s dolní hranicí vlnové délky (nejkratší možná vlnová délka), která je určena teplotou záření. Neperiodický impuls lze Furierovou transformací převést na spojité spektrum. Takovým neperiodickým impulsem může být rozpad sterilního neutrina. Protože sterilní neutrino je vyšším kvantovým stavem, avšak s nižší energií není mu dovoleno přecházet na nižší kvantové stavy, ale může existovat určitá pravděpodobnost rozpadu.


Příspěvek od: Michal Fabinger
Čas: 03:43 12.02.2001
E-mail: fabinger@stanford.edu
Web: http://go.to/fabinger
... nekdy se ale spravne nazvoslovi nedodrzuje striktne, a tak je obcas mozne slyset, ze mame foton o teplote T. Neznamena to ale nic jineho, nez ze jeho energie je E=kT. (Jinak, Lubos chtel asi rict, ze Plankova krivka ma nekde pro nejakou konecnou frekvenci maximum.)


Příspěvek od: Lubos Motl
Čas: 01:54 12.02.2001
E-mail: motl@physics.rutgers.edu
Web: http://come.to/lumo/
U jedne jedine castice nema smysl mluvit o teplote, protoze to je termodynamicky pojem a ten se jako vsechny termodynamicke pojmy stane realnym az v termodynamicke limite, kdy je pocet castic velky a lze uzit statisticke fyziky.

Teplota parametrizuje fyzikalni soustavu, ktera je v tepelne rovnovaze. Fotony s danou teplotou napriklad vypadaji jako zareni cerneho telesa, intenzita svetla v intervalu frekvenci (f,f+df) je C.f^3/exp(hf/kT-1), kde C je znama konstanta. Tohle je krivka (Planckova), ktera ma nekde minimum, ale kazdopadne teplota T (v kelvinech) odpovida energii E=kT (k je Boltzmannova konstanta) a energie E odpovida frekvenci: E=hf=hbar.omega.


Příspěvek od: Vojta
Čas: 20:02 11.02.2001
E-mail: egg@atlas.cz
Asi to není úplně k věci, ale když mám foton o frekvenci f, jakou má "teplotu"?


Příspěvek od: František Lomoz
Čas: 20:09 07.02.2001
E-mail: f.lomoz@worldonline.cz
Pánové,díky za vaše souhlasné stanovisko, dodává mi chuť pokračovat. Vážím si připomínek pana Motla. K ekvivalenci energie kterou nese hmota sterilního neutrina se ještě vrátím. Jen malou poznámku. Tepelné záření neobsahuje všemožné teploty, ale všemožné vlnové délky záření odpovídající dané teplotě.
Výpočet rotačních hladin strun (hmotností neutrin), ale i klidových hmotností elektronu, mionu a tau leptonu není až tak složitý. Zde uvedu pouze dva výrazy, které jsou součástí výpočtu. Zdrojová hmota má tři složky m1, m2 a m3, které splňují rovnici m1 = h^2/(m01.R0^2.c^2), kde h je Planckova konstanta, R0 je rovnovážná vzdálenost vazby, c je rychlost světla, m01 je primární hmotnost. Zdrojová hmotnost m1, m2 a m3 je řádově stejně velká jako hmotnost skutečných částic. Z uvedeného výrazu a ze zmíněných hmot neutrin v předchozím příspěvku vyplývá ,že primární hmotnost m01, m02 a m03 je řádu 10^16 eV, při vzdálenosti vazby R0 rovnající se klasickému poloměru elektronu. Druhým výrazem je poměr energie primární hmoty k plošné energii vazby A1 = m01.c^2/(k1.R0^2), kde A1 je bezrozměrné číslo, k1 je silová konstanta. Konstanta má tři složky k1, k2 a k3, z nichž dvě jsou kladné a mají velikost řádově 10^18 N/m. Třetí složka v případě neutrin má záporné znaménko. Co to znamená? Dvě kladné složky sbalují šest rozměrů devíti-rozměrného prostoru, třetí záporná složka rozbaluje zbylé tři rozměry. Napětí ve struně má uvedenou hodnotu, která na rozměru částice odpovídá hodnotě síly asi desetině tuny (omlouvám se za tuto jednotku síly, ale je názorná). Tak pevné je vakuum. Kombinace složek napětí pak v obecnějším komplexním vyjádření vynulují vibrační módy struny, na rozdíl od klasické vibrační hladiny třeba u molekul, kde základní vibrační hladina při kvantovém čísle v=o má vždy určitou hodnotu, není nulová.


Příspěvek od: Vojta
Čas: 23:54 05.02.2001
E-mail: egg@atlas.cz
To jsem sam rad, ze jsem nemel pravdu... :-)
Panu Lomozovi se omlouvam a Lumovi diky za vysvetleni.


Příspěvek od: Lubos Motl
Čas: 23:32 05.02.2001
E-mail: motl@physics.rutgers.edu
Web: http://lumo.come.to/
Jeste panu Lomozovi. Reliktni zareni nemuze byt zadnym dusledkem rozpadu napr. sterilnich neutrin o podobne energii. Reliktni zareni je v prvni rade perfektni tepelne zareni. Kdyz si vykreslite intenzitu jako funkci frekvence, vyjde Vam naprosto perfektni Planckova krivka a.f^3/[exp(cf)-1], kde a,c jsou konstanty. Je to fakticky nejpresnejsi zname tepelne zareni, ktere se bere jako prototyp, a lide dokazou presne merit odchylky teploty v ruznych smerech. Z rozpadu nestabilniho sterilniho neutrina byste dostal uplne jine spektrum energii nez tepelne.


Příspěvek od: Lubos Motl
Čas: 23:19 05.02.2001
E-mail: motl@physics.rutgers.edu
Web: http://lumo.come.to/
Vojto, mylis se. Rotacni mod je zcela smysluplna vec, obzvlaste v kvantove mechanice molekul dulezita.

Jak se resi molekuly v kvantove mechanice? Jadra jsou mnohem tezsi nez elektrony, tak se nejprve povazuji za klasicke objekty, ktere maji pevne pozice, a resi se kvantova mechaniky jen pro vlnove funkce elektronu. Vyjdou hladiny jako funkce poloh jader (Bornova-Oppenheimerova aproximace). Potom se teprve tyto energie stavu interpretuji jako potencialni energie mezi jadry.

Puvodni elektronova hladina se rozstepi - o trochu jemnejsi rozdeleni hladin je na rotacni mody (jadra rotuji kolem sebe navzajem), tyto rotacni hladiny jsou pak jeste jemneji rozstepeny na vibracni mody. V teorii strun prispiva rotace v zasade stejne velkou energii jako vibrace.

Anomalie slunecnich a atmosferickych neutrin (a pokusy LSND a dalsi) vedou lidi k tomu, ze neutrina osciluji, k cemuz musi byt ruzne druhy pomichane a alespon jedna hmotnost musi byt nenulova. Krome modelu se tremi neutriny, ktere tvori partnery elektronu, mionu a tauonu, nekteri lide promysleji i model se ctvrtym (osamocenym) neutrinem, kteremu se rika sterilni.

Takze ackoliv mozna, Vojto, sdilime pohled na hloubku navrhu pana kolegy Lomoze, Tvoje konkretni kritika byla asi licha. Jinak konkretne na obe otazky bych odpovedel ano - ano, struny maji rotacni mody (s ruznou hodnotou momentu hybnosti apod.) a ano, je to zcela racionalni. ;-)


Příspěvek od: Vojta
Čas: 23:08 05.02.2001
E-mail: egg@atlas.cz
Koukám, že to Luboš odeslal zrovna, když já jsem psal svůj text...


Příspěvek od: Vojta Hála
Čas: 23:01 05.02.2001
E-mail: egg@atlas.cz
Web: http://egg.baf.cz
Pokusím se odpovědět sám. Ale nejsem žádný odborník, takže je možné, že se pletu...

Ad 1) Ne.
Ad 2) Ne.

Co znamená "rotační mód"? Co je "sterilní neutrino"? Tyhle termíny slyším poprvé a tuším, že to není moje neznalost...


Příspěvek od: Lubos Motl
Čas: 22:56 05.02.2001
E-mail: motl@physics.rutgers.edu
Web: http://lumo.come.to/
Zajimave povidani, pane Lomozi, ten vypocet byste asi musel upresnit, mozna tomu nekdo rozumi lepe nez ja (ja ani presne nevim, co minite presnou "energii" tepelneho reliktniho zareni - tepelne zareni obsahuje vsemozne teploty - a proc by to melo souviset s hmotou neutrin (proc by se tedy ty hmoty mely menit s casem). Jinak ano, struny nejen vibruji, ale i rotuji. Strune lze pridavat "excitace", ktere jednak pridavaji energii, ale krome toho take obecne pridavaji moment hybnosti. Jedna z prvnich rovnic, ktere se psaly na konci 60.let v dualnich modelech (puvodni nazev stare teorie strun), byl vztah pro maximalni moment hybnosti povoleny pro hmotu "m", Jmax=m^2.alpha', kde alpha' je konstanta (Reggeho sklon).


Příspěvek od: František Lomoz
Čas: 21:55 05.02.2001
E-mail: f.lomoz@worldonline.cz

Příspěvek pro diskusní fórum teorie superstrun.
Zaujala mne citovaná otázka Stephena Hawkinga v úvodu fóra. Proč se vesmír obtěžuje existovat? Moje odpověď: protože NIC není stabilní. Pojem NIC zahrnuje nic z mnoha věcí, a také, nic samo o sobě, třeba bod vakua. V tomto podání má každé nic šanci zrodit vesmír. Vysvětlím-li podstatu bodu vakua, pak chápu stvoření vesmíru i za předpokladu, že velikost Boha spočívá v jeho neexistenci. A nyní vážně míněná 1.otázka: Zahrnuje teorie superstrun, kde částice jsou vibračními módy strun, také rotační módy těchto strun? Sám jsem se pokusil odpovědět na tuto otázku. Rotační módy vedou k nalezení takových „struktur“ strun, kdy vibrační módy jsou „vynulovány“. Tyto rotační módy pak reprezentují jednotlivé druhy elementárních částic. Nabité a neutrální leptony, horní a dolní kvarky. Přičemž vyšší kvantové stavy jsou známé „odrůdy“ neutrina, elektronu, down kvarku a up kvarku. Tento rotačně-kvantový popis dovoluje stanovit hmoty zmíněných částic v jejich rodinách podle jednotného schéma. Např. 0,069 eV; 0,041 eV; 0,078 eV a 0,000235 eV pro hmoty elektronového, mionového, tauového a sterilního neutrina. Přitom hmota sterilního neutrina zde vystupuje jako třetí rotační stav ve druhé rodině neutrin. Tři zdrojové hmoty, vystupující v rotačně-kvantovém popisu neutrin, mají zajímavou vlastnost. Součet hmoty sterilního neutrina se zdrojovými hmotami je nulový. Přitom samotná hmota sterilního neutrina je ekvivalentní energii záření o teplotě 2,73 K. Je reliktní záření popelem Velkého třesku nebo zbytkem po rozpadu sterilních neutrin? Vždyť každé nic se rozpadá. Takže, 2. otázka: Může být v odpovědi na 1. otázku něco racionálního?


Příspěvek od: Lubos Motl
Čas: 19:43 28.01.2001
E-mail: motl@physics.rutgers.edu
Web: http://lumo.come.to/
A to si jeste uvedom, Vojto, ze se Te to vlastne az tolik netyka: jak teprve museji byt netrpelivi ti, kteri s tim neco maji do cineni. ;-) Kniha uz je vytistena na stranky, jeden z vytisku pani redaktorka finalne kontroluje, jeden cestuje postou ke kontrole prekladateli a Mirek Belan pracuje na rejstriku. Moznost, ze prijede Brian Greene do CR, je jen o trosinku male pravdepodobnejsi nez predtim.

Tipuji, ze z MF tohle forum nikdo necte ;-), tolik si zase never. Mozna ctou v praci noviny? Tohle nakladatelstvi smeruje ke kolapsu, vetsina redaktoru utekla (redaktorka pani Amelova pro Elegantni vesmir s vetsinou sla do Paseky za Pistoriem). Asi bychom meli byt radi, ze alespon v tomhle ctvrtinovem tempu nakonec snad knihu vydaji. ;-)


Příspěvek od: Vojta Hála
Čas: 12:51 28.01.2001
E-mail: egg@atlas.cz
Mám tradiční otázku: Kdy vyjde Elegantní vesmír? Hodně lidí se mě na to ptá a já, po roce čekání, začínám být značně netrpělivý. Už je aspoň obálka? Také se mluvilo o tom, že Brian Greene přijede do Prahy, to už zřejmě neplatí... Čte tohle fórum někdo z Mladé fronty?


Příspěvek od: Luboš Motl
Čas: 01:21 15.01.2001
E-mail: motl@physics.rutgers.edu
Web: http://come.to/lumo/
Teorie strun omezuje, jaký tvar mohou mít svinuté rozměry. Časoprostor musí splňovat rovnice pohybu a ve vakuu jsou to přibližně Einsteinovy rovnice, Ricciho tenzor musí být roven nule, tytéž rovnice, které zodpovídají za gravitační sílu podle Einsteinovy obecné relativity, ale teď musí platit i pro indexy odpovídající svinutým směrům. Taková rovnice zásadně omezuje, jaké tvary mohou variety mít. Kromě toho ale lidé promýšlejí přednostně modely, které zachovávají určitou supersymetrii. Pokud je tvar příliš komplikovaný, supersymetrie úplně zmizí. Jestliže ale supersymetrie zbude, ve většině případů z toho plyne, že rovnice pohybu jsou splněny. Šestirozměrné variety, které zachovávají při svinutí heterotické teorie alespoň nějakou supersymetrii ve čtyřrozměrném prostoru, se nazývají Calabi-Yauovy variety, a tvoří podmnožinu prostorů - variet, které splňují rovnice pohybu. Přesto je jich stále příliš: přibližně deset tisíc různých topologických tvarů a pro každý topologický tvar existuje "modulární" prostor konečné dimenze (někdy třeba 102, jindy více či méně), který udává možné plynulé deformace. Teorie strun umožňuje procesy, které mění topologii, čili od libovolného Calabi-Yauova prostoru se asi lze dostat k libovolnému jinému - modulární prostory jsou spojeny do jednoho (teorie strun je jedna i v tomto významu). Kromě toho na modulárních prostorech vzniká po narušení supersymetrie potenciál (funkce, udávajíci potenciální hustotu energie) a opravdové vakuum musí sedět v lokálním minimu, kterých by mělo být jen omezené množství. V principu tedy očekáváme, že jakmile budeme přesně vědět, jak počítat potenciál apod., tvar svinutých rozměrů a z něho plynoucí fyziku bude možné určit v podstatě jednoznačně. Dnes takovou přesnou definici ale stále neznáme.


Příspěvek od: Vojta Hála
Čas: 01:00 15.01.2001
E-mail: egg@atlas.cz
Web: http://egg.baf.cz
Do jaké míry je tvar variety volitelným parametrem strunových teorií?


Příspěvek od: Michal Fabinger
Čas: 19:24 15.12.2000
E-mail: fabinger@stanford.edu
Web: http://mbox.troja.mff.cuni.cz/~mfab5099/struny.html
Proste Lorentzova transformace je, kdyz prejdu do pohybujici se soustavy, jejiz pocatek souradnic splyva s puvodnim pocatkem souradnic v case t=0. Treba kdyz se budu na svet divat z jedouciho vlaku, a navic budu libovolne otoceny vuci smeru puvodnich souradnic. Posunuti je, kdyz zmenim polohu pocatku souradnic. Poincareho transformace je potom transformace slozena z Lorentzovy transformace a posunuti.


Příspěvek od: Lubos Motl
Čas: 16:25 15.12.2000
E-mail: motl@physics.rutgers.edu
Web: http://lumo.come.to/
Poincareho transformace je libovolna transformace Poincareho grupy. To jsem moc nevysvetlil, ze ano? ;-)

Poincareho transformace je slozeni libovolneho posunuti souradnic (v libovolnem ze 3 prostorovych rozmeru nebo v case) a nejake Lorentzovy transformace.

Lorentzova transformace je prvek Lorentzovy grupy. To jsem moc nevysvetlil, co? :-)

Lorentzova grupa obsahuje vsechny mozne rotace 3 (nebo obecne vice) dimenzi plus vsechny "boosty", coz jsou transformace michajici cas a prostor, odpovidajici prechodu k jine vztazne soustave (pohybujiciho se pozorovatele).

Poincareho grupa ma tedy deset generatoru, P0,P1,P2,P3,J01,J02,J03,J23,J31,J12. P jsou translace (posunuti), J0i jsou boosty - ktere se obvykle nazyvaji Lorentzovy transformace - a Jij jsou rotace (zde i,j=1,2,3), ktere ovsem do Lorentzovy (a proto i do Poincareho) grupy patri tez.


Příspěvek od: Pavel Mikulka
Čas: 10:38 15.12.2000
E-mail: mikulka@nspuh.cz
Mohl by mi někdo trochu přiblížit, o co je to Poincareho transformace ?


Příspěvek od: Lubos Motl
Čas: 16:57 09.12.2000
E-mail: motl@physics.rutgers.edu
Web: http://lumo.come.to/
K tomu se da rict nekolik veci. Predne, otazka, proc to byla "prave hmota a ne anihmota" je nefyzikalni, protoze je jen konvence, ktera je ktera (pricemz je logicke, ze tu, ze ktere jsme sami, nazyvame hmotou, zatimco tu druhou, ktere je jako safranu a kterou je treba pracne vyrabet na urychlovacich, nazyvame antihmotou).

Je to trochu jako s tim Feynmanem u vojenskeho psychologa, ktery se zepta, jake ma Feynman rad cislo. Feynman rekl "64", protoze takova otazka byla pro Feynmana samozrejme totalni hovadina. Psycholog se zeptal, proc prave 64 a ne treba 73, a Feynman rekl, ze kdyby byval rekl "73", psycholog by se zeptal stejne.

V ranych stadiich vesmiru bylo hmoty i antihmoty mnohem vice, ale vetsina zanihilovala a zbyla asi tak jen ta miliardtina hmoty puvodniho mnozstvi.

Zakony prirody zachazeji s hmotou a antihmotou skoro stejne, ale nikoliv uplne stejne (stejne tak prirodni zakony nejsou perfektne zrcadlove symetricke, napriklad neutrino se vzdy toci doleva jako levotocivy sroub), ale kazdopadne lze tuto nerovnovahu vysvetlit fyzikalne.


Příspěvek od: Vojta
Čas: 16:16 09.12.2000
E-mail: egg@atlas.cz
Nedavno se me nekdo ptal, proc je ve vesmiru spousta hmoty a skoro zadna antihmota. Myslim, ze pokud se vyskytne nerovnovaha v pomeru hmoty a antihmoty, musi dojit ke zpetne vazbe a situace zustane nesymetricka definitivne. Ale proc se jazycek vah spustil prave na stranu hmoty a ne na druhou? Je to zpusobeno nejakou nahodou v prvnich fazich vyvoje vesmiru anebo existuje fyzikalni vysvetleni (napr. ve strunove kosmologii) ?


Příspěvek od: Lubos Motl
Čas: 19:47 06.12.2000
E-mail: motl@i.am
Web: http://lumo.come.to/
Jo, souhlasim se vsim, v cem me kdo opravil. Kosmicka struna (dana monodromii, akci, ktera nastane, pokud strunu obejdeme) neni totez jako kosmologicka struna (jednorozmerna site mohou prispivat k hustote vesmiru podobnym zpusobem jako kosmologicka konstanta apod.).

M-teorie neobsahuje struny v tom smyslu, ze obsahuje jen dvourozmerne membrany a petirozmerne petibrany. Svinutim jednoho rozmeru M-teorie na kruznici ziskame superstruny typu IIA, kde vlaknite jednorozmerne objekty v teorii superstrun typu IIA jsou ve skutecnosti membrany M-teorie, jejichz jedna dimenze je napjata kolem kruhove dimenze.

M-teorie v sirsim smyslu slova je slozita prisera, ktera v ruznych limitnich podminkach muze vypadat jako ruzne teorie superstrun, v nichz jsou nejlehci utvary jednorozmerne vlaknite objekty (struny), ale ma take 11-rozmerny limit, ve kterem ziji gravitony, membrany, petibrany, ale zadne struny. Zatimco v 80.letech byly struny vzdycky vnimany jako fundamentalni, dnes je teorie obecnejsi v tom smyslu, ze ruzne druhy strun jsou nejdulezitejsimi objekty jen v ruznych limitach, ale obecne M-teorie (v sirokem smyslu slova) demokraticky obsahuje objekty vsech moznych dimenzi...


Příspěvek od: Vojta Hala
Čas: 19:10 06.12.2000
E-mail: egg@atlas.cz
Web: http://egg.baf.cz
Fabi, muzes to trosku rozvest pro neznale? V jakem smyslu M-teorie neobsahuje jednodimenzionalni struny...


Příspěvek od: Milan Simkovic
Čas: 07:38 06.12.2000
E-mail: msimk@pobox.sk
Nemyslel som kozmicke struny, ale vlaknovitu strukturu rozlozenia tmavej hmoty - blizsie informacie napr. v casopise Kozmos 3/2000.
Preco ta hmota nie je rovnomerne rozptylena (ako prach) s gulovo symetrickymi zhusteninami sposobenymi lokalnou gravitaciou, tak ako na mensich skalach ?


Příspěvek od: Michal Fabinger
Čas: 01:45 06.12.2000
E-mail: fabinger@stanford.edu
Web: http://mbox.troja.mff.cuni.cz/~mfab5099/struny.html
Ja osobne bych rekl, ze tohle vlaknite rozlozeni hmoty s jednorozmernosti superstrun nic spolecneho. Navic si myslim, ze ta skutecna teorie, podle ktere se ridi priroda, nemusi vubec zadne struny obsahovat. Treba jako jedenactidimenzionalni M-teorie.


Příspěvek od: Lubos Motl
Čas: 17:38 05.12.2000
E-mail: motl@physics.rutgers.edu
Web: http://come.to/lumo/
Sorrac, poslal jsem tu blbost drive, nez jsem mel cas ji zkontrolovat. Holonomie melo byt "monodromie". ;-) Za pripadne dalsi chyby se omlouvam, snad nekdo napise neco smysluplnejsiho.


Příspěvek od: Lubos Motl
Čas: 17:35 05.12.2000
E-mail: motl@physics.rutgers.edu
Web: http://come.to/lumo/
Asi bych odpovedel Ne, samotna existence kosmickych strun neni dukazem teorie strun.

Z druhe strany ta Vase argumentace je smysluplna a Edward Witten, asi nejuznavanejsi odbornik v oboru, povazuje nalezeni otisku strun, ktere se roztahly do kosmologickeho meritka, za podivinsky, ale jeho nejoblibenejsi scenar, jak jednou dokazat teorii strun, protoze nic by tak jasne otazku nevyjasnilo.

V kosmologii se da uvazovat o vlaknitych objektech, ktere zpusobuji chybejici uhel (celkem kolem sebe nemaji 360 stupnu, ale mene). Pokud obejdeme kosmickou strunu, muze se neco stat (napriklad se muze zmenit hmota na antihmotu apod.), tomu se rika holonomie.

Kosmicke struny v teorii strun hraji take svou ulohu, ale nejsou to nutne tytez struny.


Příspěvek od: Milan Šimkovič
Čas: 11:27 05.12.2000
E-mail: msimk@pobox.sk
Nejaké je to tu mŕtve, preto sa to tu pokúsim oživiť nasledujúcou otázkou: Je možné považovať rozloženie hmoty vo vesmíre na veľkých škálach (stovky megaparsekov) za dôkaz strunovej teórie?
Vysvetlím: Astronomickým pozorovaním sa zistilo že rozloženie hmoty na veľkých škálach má vláknovitú (= strunovitú) štruktúru, zatiaľ čo na menších škálach (nadkopy, kopy galaxií, galaxie, hviezdy ...) je rozloženie hmoty sféricky (alebo dôsledkom rotácie cylindricky) symetrické.
Ďaľšou všeobecne akceptovanou kozmologickou teóriou je inflačná teória. Jedným z jej dôsledkov je, že oblasti vesmíru ktoré boli pôvodne veľmi blízko (boli v kauzálnom spojení) po inflácii boli "roztiahnuté" do veľkých rozmerov.
Struny môžeme interpretovať ako jednorozmerné fluktuácie energie oproti pozadiu.
Nie je možné veľkorozmerné vlákna (fluktuácie hustoty) hmoty považovať za dôsledok infláciou "roztiahnutých" malých fluktuácií energie mikroskopických strún ?


Příspěvek od: Michal Fabinger
Čas: 23:02 15.11.2000
E-mail: fabinger@stanford.edu
Web: http://mbox.troja.mff.cuni.cz/~mfab5099/struny.html
Cau Vojto!

Kdyz mas ve fyzice nejakou spojitou symetrii, vzdycky se s ni poji nejaka zachovavajici se velicina. Napriklad, kdyz mas homogenni prostor, zachovava se ti hybnost. Pokud mas symetrii vuci posunuti v case, zachovava se ti energie. Tyhle dve veliciny spolu tvori ctyrvektor, a ty muzes tedy rict, ze mas ctyri symetrie. Supersymetrie je zvlastni symetrie, kde zachovavajici se velicinou neni ani vektor, ani zadny skalar, ale spinor. Pocet komponent vsech zachovavajicich se spinoru je proste pocet supersymetrii, ktere mas. Napriklad v M-teorii a ve strunach typu IIA a IIB jich mas 32. V heterotickych strunach a ve strunach typu I jich mas jenom polovinu - 16. Kompaktifikace muze zpusobit, ze nektere z techto velicin se uz nebudou dale zachovavat. Pokud se ti prestanou zachovavat tri ctvrtiny puvodnich komponent onech spinoru, reknes, ze ti zbude ctvrtina puvodni supersymetrie.

Zdravi Michal


Příspěvek od: Vojta
Čas: 08:22 15.11.2000
E-mail: egg@atlas.cz
Diky, uz je to o neco jasnejsi.
Jeste bych rad vedel, co znamena "zustane nam ctvrtina supersymetrie"...


Příspěvek od: Lubos Motl
Čas: 00:40 15.11.2000
E-mail: motl@physics.rutgers.edu
Web: http://lumo.come.to/
Zdravice!

Kdyby mel nekdo nejaky lepsi obrazek na obalku, nez navrhla MF, pripadne zpusob, jak ho vyrobit (a souvisi se strunami), tak at se ozve! ;-)

Teorie strun nedovoluje, aby svinute rozmery mely jen tak jakykoliv tvar. Tento tvar musi splnovat jiste rovnice, nezjednodusim prilis, kdyz reknu, ze Einsteinovy rovnice. Pokud ve smeru skrytych rozmeru nejsou zadna magneticka pole apod. a vazebna konstanta je opravdu konstanta, musi byt Ricciho tenzor nulovy.

To dramaticky omezuje mnozstvi tvaru, do nichz lze skryte rozmery svinout. Lide se vetsinou zajimaji o supersymetricka vakua, ktera maji mnoho zajimavych vlastnosti. K tomu, aby nam zbyla pri svinuti rozmeru nejaka supersymetrie, je treba jeste vice specialniho tvaru.

Neodpustim si trochu matematiky: pokud paralelne presuneme vektor v 6 dimenzich podel nejake krivky vedouci skrytymi rozmery, tento vektor se obecne otoci rotaci z O(6). Pokud je varieta orientovana, tak to musi byt v podgrupe SO(6).

Grupa SO(6) je v podstatne izomorfni SU(4) a SU(4) ma podgrupu SU(3). Pokud dovolime jen takove variety, ze se vektor pri objizdce muze otocit jen nejakym prvkem v SU(3) podgrupe grupy SO(6), potom nam ctvrtina supersymetrie zustane. Varieta - tvar svinutych dimenzi - ktery takto omezuje "holonomii" (mozne rotace, ktere vektor zmeni, pokud ho projedeme po nejake krivce) - se nazyva Calabi-Yauova varieta.

Calabi-Yauovych variet je porad celkem hodne, ale je jich mnohem mene, nez variet, ktere splnuji Einsteinovy rovnice, a tech je jeste o hodne hodne mene, nez je mnozstvi vsech variet.

Mnozina vsech moznych Calabi-Yauovych tvaru je podle vseho sjednoceni "modularnich prostoru" ruzne dimenze, mezi nimiz lze navic cestovat tim, ze zmenime topologii.

No neni tu asi dost mista, aby to slo vysvetlit poradne, ale v EV je prece jen mista vice. ;-)

Zdravi
Lubos


Příspěvek od: Vojta Hála
Čas: 00:29 15.11.2000
E-mail: egg@atlas.cz
Web: http://www.vojtisek.com
Přátelé,
chtěl bych vás přerušit. Pojďme zpět ke strunám.

Už je to delší čas, co jsem tu položil otázku, jak se Calabi-Yauovy variety liší od ostatních. A proč právě ty jsou nejvhodnější pro strunové teorie. Asi je těžké odpovědět bez matematiky.. Napadá někoho, jak to popsat?

Dobrého dne!


A Předchozí stránka
Připomínky, dotazy či návrhy směrujte sem, příspěvky do diskuse sem.CNW_Counter
Můžete se vrátit zpět na homepage, na superstruny anebo tam, odkud jste přišli.